Você está na página 1de 291
Miller Dias de Araujo Os Segredos da Algebra para IME ITA Olimpiadas Editora Vestseller FORTALEZA - CE 41? Edigdo Julho/2018 Apresentagao Escrever um livro nao é 80 expor nele a matéria e resolver as questées selecionadas. E, antes, relacionar contetidos, conjugando didaticamente ideias e métodos eficazes a0 longo da obra. Trata-se de um trabalho arduo, pois sao varios og aspectos (muitas vezes sobrepostos) a abordar, e somente quem € autor sabe a dificuldade de levar tudo isso a um bom termo. Por outro lado, a escrita deste livro ofereceu-me novas experiéncias ¢ oportunidades de amadurecimento tanto como professor quanto como autor. Além disso, 0 fato de haver poucos, talvez pouquissimos, livros hoje no mercado que detalham os assuntos de que trato aqui fez com que minha motivagao fosse ainda maior. Nos oito primeiros capitulos, exponho toda a teoria, de forma detalhada e demonstrada. Trato do assunto e, apds fazé-lo, trago exemplos resolvidos para facilitar a compreensao. Na sequéncia, ha uma bateria de exercicios para oleitor praticar os assuntos fixar 0 que aprendeu. Vocé vera que a maioria das questées tem mais de uma resolucao, & algumas delas foram repetidas em capitulos diferentes. Isso se da para que vocé possa, se quiser, resolvé-las com 0 assunto daquele novo capitulo, aumente sua visdo geral dos contetidos e afie ainda mais seu raciocinio para utilizar em exercicios futuros. Nos dois ultimos capitulos, concentrei os gabaritos, as sugestdes € as resolugées, para que o leitor possa conferir suas proprias respostas e, possivelmente, aprender ideias novas também a partir das resolugdes. Miller Dias De Aratjo Prefacio No século XVII, 0 mateméatico francés Jean Le Rond D’Alembert afirmou “A algebra € generosa: frequentemente ela da mais do que se Ihe pediu". D'Alembert, que Possui importantes descobertas na algebra, como o Teorema Fundamental da Algebra, ndo poderia ter sido mais feliz em sua célebre frase. A algebra é a ferramenta mestra Para o desenvolvimento de todas as ciéncias naturais. O que seria da fisica sem a Algebra? O que seria das outras areas da matematica sem a algebra? No nosso pais, de forma geral, as pessoas menosprezam a importancia da matematica em suas vidas. Mal sabem essas pessoas 0 quanto elas seriam mais felizes se soubessem, por exemplo, calcular 0 consumo mensal de cada aparelho eletronico de sua residéncia ou calcular o consumo de combustivel de seu automdvel em faixas diferentes de velocidade. Para tanto, bastaria que tivessem uma base mais sdlida de algebra e aritmética. E essa base mais solida em matematica aprende-se com bons professores e com bons livros. No Ambito dos concursos militares e olimpiadas cientificas, uma base sdlida em matematica nado é uma op¢ao, é uma obrigagao. Ha um detalhe importante sobre o ensino dos primeiros conceitos de algebra: eles so abordados no ensino fundamental, momento em que nado ha espacgo para aprofundamentos, fazendo com que os livros apresentem a algebra de maneira superficial. Nos vestibulares tradicionais (ENEM e vestibulares de universidades federais, estaduais e particulares), nado ha uma exigéncia muito grande quanto a algebra. Porém, nos concursos militares e olimpiadas de matematica, a algebra é exigida em um nivel muito alto, sendo necessério um estudo mais aprofundado. De modo a auxiliar nesse aprofundamento, a obra “Os Segredos da Algebra Para IME/ITA/OLIMPIADAS”, do eminente autor Miller Dias De Aratijo, 6 perfeita. Todos os tépicos da algebra fundamental estao presentes, incluindo potenciagao, radiciagdo, produtos notaveis e fatoragao. Em cada capitulo, a teoria algébrica 6 apresentada de forma detalhada, sequida de muitos problemas resolvidos. Varias identidades algébricas séo demonstradas, algumas pouco conhecidas. Certamente, a cereja do bolo sdo os exercicios propostos, em grande quantidade e de excelente qualidade, a maioria retirada de concursos militares e olimpiadas de matematica de todo o mundo. Vocé vai se deparar com exercicios do Colégio Naval, ITA, IME e também de olimpiadas de diversos paises. Deve-se destacar a organizagao dos capitulos, nos quais os conceitos mais fundamentais sdo apresentados depois constarem os topicos mais avangados, facilit leitor. O ultimo capitulo do livro é reservado para exercicios propostos. A presente obra é um verdadeiro tesouro, indispensavel Para qualquer Pessoa que deseja formar uma base sdlida em algebra. Recomendo ao leitor que resolva esse livro de capa a capa, prestando atengdo nas nuances de cada passagem algebrica. Existe muita poesia, arte e Magia nessa obra prima do Miller Dias. Parabéns ao grande mestre! Tenho que admitir que, durante a leitura do livro, me perguntei o quanto minha vida de concurseiro militar teria sido mais facil se eu tivesse, a época, um livro de to alto nivel para ajudar minha base algébrica. Certamente esta obra de Miller Dias De Araujo sera uma importante ferramenta Para todos formarem uma base mais solida em algebra. primeiro, Para somente lando o entendimento do @S SolugGes e dicas dos Marcelo Rufino de Oliveira Agradecimentos A Deus, pelo dom da vida. Aos meus pais, que sempre me apoiaram e me incentivaram em todas as minhas caminhadas. A meus amigos, em especial Thais Pereira Soares e Johnatan Bruno, pelo apoio, motivagao e incentivo; Meus professores, que ensinaram-me com muita dedicacao. Ao professor Renato Brito, pelas criticas e sugestées. Ao professor Marcelo Rufino pela escrita do prefacio. Ao professor Claudio Neves pelas revis6es. Ao professor Luiz Vieira dos Santos, pelos ensinamentos, pela paciéncia e por ter-me iniciado no caminho da Matematica IME ITA. Sumario Capitulo 01: Potenciacao 4.1) Definigao. ) Produto de Poténcias de jesma Base. ) Divisdo de Poténcias de Mesma Base. ) Poténcia Elevada a Poténcia ) Produto Elevado 4a Mesma Poténci: ) Poténcia Elevada a Poténcia, n vezes . ) Poténcia de Ordem Superior. ) Poténcia com Expoente em PG. ) 0 1 Poténcia com Expoente Negativo. )) Divisao Composta 2 3 4 5 6 7 8 9 1 41) Poténcia com Expoente Fracionario. 4 4 1 1 1 1. 1 1 1 4; Capitulo 02: Radiciagao 2.1) Definigao............ 2.2) Produto de Radicais ‘de Mesmo indice. 2.3) Divisdo de Radicais de Mesmo indice. : 4) Raiz de uma Raiz........... 5) Produto de Radicais de indices Di 6) Raiz de Fragao Composta... 7) Séries Finitas de Radicais....... 2.8) Séries Infinitas de Radicais 2.9) Divisao Composta Infinita. 2.10) Radicais em Cadeia Infinita, 2.11) Operagdes com Radicais. Capitulo 03: Racionalizagao 3.1) Quocientes Notaveis... 3.2) Fator Racionalizante 3.3) Radicais Dupios....... 3.4) Topicos Avangados em Radicais Duplo Capitulo 04: Expressées Algébricas 4.1) Tipos de Expresses Algébricas. 4.2) Valor Numérico...... 4.3) Operagdes com Expressoes. Algébricas. 93 405 124 125 126 427 Capitulo 05: Produtos Notaveis 5.1) Quadrado da Soma de Dois Termos......... 130 5.2) Quadrado da Diferenga Entre Dois Termos.. 131 5.3) Identidade de Legendre para a SOMA... 132 5.4) Identidade de Legendre para a Diferenga.......... : : a 133 5.5) Identidade de Lagrange para a Soma 133 5.6) Identidade de Lagrange para a Diferenca. 134 5.7) Produto da Soma pela Diferenga... 140 5.8) Identidades de Stevin. 142 5.9) Cubo da Soma de Dois Termos. 149 5.10) Cubo da Diferenga de Dois Termos. 149 5.11) Identidade de Cauchy.... 152 5.12) Quarta Poténcia da Soma e da Diferenga.. 157 5.13) Identidades de Legendre. 158 5.14) Quinta Poténcia da Som 160 5.15) Quinta Potencia da Diferenga. 161 5.16) Identidades para Termos Reciprocos. 165 5.17) Quadrado da Soma de Trés Termos.. 173 5.18) Identidade de Lagrange Para Trés Termos. 175 5.19) Produto Dois a Dois Elevado ao Quadrado.. 176 5.20) Identidades de Argand... 181 5.21) Quadrado da Soma de Quatro Termo 182 5.22) Cubo da Soma de Trés Termos... 183 5.23) Identidade de Gauss... 185 5.24) Soma de Quatro Termos Elevado ao Cubo.. 186 5.25) Quarta Poténcia de Trés Termos. .. 187 5.26) Identidades de Stevin para Trés Termos 189 5.27) Identidade de Sophie-Germain......... 191 5.28) Identidade de Chrystal... 194 5.29) Identidades Condicionais. 199 5.30) Topicos Avangados em Produtos Notavei 211 Capitulo 06: Fatoragao 6.1) Critérios de Fatoragao.. 213 6.2) Agrupamento ou “evidénci 213 6.3) Quocientes Notaveis 214 6.4) Completando o Produto Notavel 216 6.5) Cruzadinha Simples.... 226 6.6) Teorema do Fator ou das Raizes Racionai 230 6.7) Fatorando Polinémios do 3° Grau. - 231 6.8) Cruzadinha Dupla.... 234 6.9) Cruzadinha Dupla Especial 238 6.10) Fatorando Polinémios do 5° Grau. 243 6.11) Cruzadinha Tripla... 6.12) Topicos Avancados ef em m Fatoracao Capitulo 07: Polinémios Simétricos 7.1) Forma de um Polinédmio Simétrico.... 7.2) Propriedades dos Polinémios Simétricos. 7.3) Fatoragao por Polinémio Simétrico. 7.4) Polinémio Alternado... 7.5) Propriedades dos Polinémios ‘Alternados 7.6) Fatoragao por Polinémio Alternado......... Capitulo 08: Somas de Newton 8.1) Somas de Newton para Dois TermoS..........00.cccccceceeeeeseeseees 212 8.2) A Notacao Sigma. 3) Somas de Newton para Trés Termo: 4) Generalizag&o para um Polinémio de Grau n Capitulo 09: Respostas e Sugestées Capitulo 10: Resolugées Bibliografia Os Segredos da Algebra para IME/ITA/OLIMPIADAS 1 Capitulo 01 - Potenciacao Introdugao A determinagdo da poténcia de um numero é feita pela multiplicagdo de fatores iguais. O expoente possui um papel fundamental na potenciagao, pois ele & quem define quantas vezes a base sera multiplicada por ela mesma, Vejamos como trabalhar com essa ferramenta importantissima na resolugéo de problemas. 1.1) Definigao: Dado um numero real “a” qualquer, tomemos n como um numero natural. O produto de n fatores “a" é igual 4 enésima poténcia de “a”. “n" vezes a-a-...a=a"|; VneNeacR. Consequéncias da definigao: a) [a2 =1; vac R-{O,o}. c) [OP =O}; vac R-{0, x}. b) [atzal; vaeR. a) [P=d; vaeR-{x}. Exemplos de Aplicagao 01 a) 2001-1. d) o'785=0. by P=1. e) of-o. 1 (1750234 _ 4 c) (40967?) - 409677. ) 4 1.2) Propriedades das Poténcias: TT , nite A seguir, veremos as propriedades mais importan iremos resolver varios problemas usando 0 minimo de calculo algébrico. Vamos lal s das poténcias. Com elas, P41. Produto de Poténcias de Mesma Base: : Num produto de poténcias de mesma base, repete-se a pase e somam-se 0: expoentes. 12 1 Potenciagaéo Demonstraca "m" vezes "vezes "m+n" vezes a™ a" =a.a--a-acay.a = a™ — =a-aaa.a a™'"|; vm,neNeaeR. Exemplos Resolvidos 04 a) 8.5% -59- 18 53.515 518 a by a a a oo ag gp, c) m?.m7P =m2P+7P <> m2P.m7P =m. P2. Divisao de Poténcias de Mesma Base: Numa diviséo de poténcias de mesma base, repete-se a base e subtraemwe os expoentes. Demonstraca “m" vezes 1 “m—n" vezes aaa: aaaa-.. in" vezes vm,neN, ac Rea" <0. 30 a) 2 12 3° ae 12 > 32 . 3 32 16 b) 2 = 22016-1008 _, 27018 1008. 21008 21008 100 cy) X= 100. 72 28 | x Os Segredos da Algebra para IME/ITA/OLIMPIADAS, 13 P3. Poténcia Elevada a Poténcia: Numa potéi de uma poténcia, repete-se a base e multiplicam-se os. expoentes. Demonstracao: a" "|; vmneNeaeR. Exemplos Resolvidos 03 3) (ry p53 (725) =778 by (p28Y" = pm .. (p20) =p? P4, Produto Elevado 4 Mesma Poténcia: ‘Num produto elevado 4 poténcia, eleva-se cada base ao expoente. [aby =a" oh Demonstracao: jezes, (a-b)" =(a-b)-(a-b)-... zes_"n" vezes a-b-b...-b (a:b) @ (ab? . (a-b) =a -b"|; vneN e abeR. 14 1 Potenciagaéo Consequéncias: roduto de varios termos, elevado a potén ‘O produto de varios termos elevado a poténcia é igual ao produto de cada tem elevado a poténcia. mo (abc...) =a"-b"-o", Demonstragao: (a-b-c...)" =(a-b-c...)-( (a-b-c...) "n"vezes "n"vezes “n" vezes © (a-b-c...)" =a-a-.-a-b-b- (a-b-c...)" =a" -b"-0"..]; vneN e@a,b,c,...eR Exemplos Resolvidos 04 a) (p-mjm* =(py™* (my +. (pe) = pA mr b) (a-b-0)*® =(a)*? -(b) -(b)?~. (a-b-6) a2 . p25 025. 02) Se as poténcias tiverem expoentes: Generalizando a consequéncia anterior para produto de poténcias, tem-se: Demonstracao: , , "n" vezes (a bY .c? -) (a bY ?....).(a% bY -0?...):. “n" vezes “n" vezes = (a*-bY oJ =a¥ a. a® bY bY .....b¥ 7 -c?-. (2% “bY -c” “n" vezes eNeap,c,...eR Os Segredos da Algebra para IME/ITA/OLIMPIADAS 15 Exemplos Resolvidos 05 a) (p*9 a) yr =(p"8 pr (msn) «(p's mo" yp =p 842M pytOnem by (a b¥cF) =(a*P (bY) (02) (at pt ct) 2 pv 02 Problemas Propostos Questao 1.1 (AHSME-1952) Qn 4 2 (2 ) ——————. , quando simplificado, é: 2. (2) ay 2" -3 b) -2"1 o) 1-2" at e) BIN Questao 1.2 Danan (G ap \2 y Questdo 1.3 ze gps? aed Determine (2 3° 7 . Questdo 1.4 (AHSME-1971) _ O numero 247+) 2-1) 4. 2° 6 igual a: a) 2% by 221) °) (21) do e)2 Questao 1.5 404.1518 .3311.7717 3413 Simplifique FO 7g ap 1 = 1 Potenciagao 16 Questao 1.6 nt nt Simplifique gi +2 Brn ote Questao 1.7 Simplifique pag M2 yak nk Questao 1.8 nk aa ee = x"vezes x*vezes k . kook Simplifique x" +x" +...+x" xa XM exe xk xk, Questao 1.9 b3vezes a®vezes Reduza a® +a? +...+a°-bP+b?+...+b?. Questao 1.10 b®vezes, aPvezes Reduza a?.a®.....a® .b?-b?.....b?, Os Segredos da Algebra para IME/ITA/OLIMPIADAS. 17 P5. Poténcia Elevada a Poténcia, Elevada a Poténcia “‘n” Vezes: ‘Numa poténcia elevada a poténcia, elevada a poténcia n vezes, o expoente 6 dado pelo produto de poténcia de mesma base. pvmneNeacR "nn vezes flor?) | “n" vezes mm... Exemplos de Aplicagao 02 "100" vezes in” vezes 5 Pema [let -# a) (ey "x" vezes. "2017" vezes c(i lef P6. Poténcia de Ordem Superior: ‘Numa poténcia dé order superior, repete-se a base é eleva-se 0 expoente a poténcia. "p" vezes [ot] | a ;vmnpeNeacR 18 1 Potenciagao Demonstraca' ae n |{ er} | sme Iv ) nenen a =a™ = Consequéncias: 1) Se as poténcias forem distintas: Se as poténcias forem distintas, repete-se a base e multiplicam-se os expoentes distintos. . “| vmnp,...eNeaeR. Exemplos de Aplicagao 03 nt vezes_ ( 8 ( y Nes i 2". 3P.58 {ion\) | 2 lel) on a) | | a) 2 a \ "m" vezes Z oe (apy bre (eel | a) || (a bY by | I j ( J 2) Se os expoentes forem distintos: Se as poténcias forem distintas e seus respectivos expoentes forem iguals podemos usar a propriedade do produto elevado a poténcia. :vm,n,p,...eNeaeR. Os Segredos da Algebra para IME/ITA/OLIMPIADAS 19 Demonstraca f | =a phn Exemplos Resolvidos 06 \r ( 3 ° | g2 3-5-7" | (gr | | _ gat | \ \ ) 42)" a \(4z)" | b) wr pr = plavael io" |< ploxvay" 3) Se os expoentes forem distintos e suas bases i ig Se as poléncias forem distintas e suas respectivas bases forem iguais, podemos usar a propriedade do produto de poténcia de mesma base. ivmng..eNeaeR.. 20 1 Potenciagao Exemplos Resolvidos 07 60 Ss { 0 riod 7 ie y wg 80 lerP | (\ ( x* { s b) ley" en ey) |e ed ed) 4) Se os expoente forem iguais e as bases iguais: Se os expoentes forem iguais e suas respectivas bases também sao iguas podemos usar 0 produto de poténcias de mesma base. “p" vezes Demonstracao: “pr vezes i #5" nym | p" vezes Pom? m™ m | =am™ mm “p" vezes mo ( ‘at y | Os Segredos da Algebra para IME/TAOLIMPIADAS a Exemplos Resolvidos 08 b) (eT) a ON, P7. Poténcia de mesma base e expoentes em PG: ‘Numa poténcia com expoentes em PG, repete-se a base, e 0 expoente expressa-se Como soma de uma PG. “n" parénieses a [ater] : cvmneNeaeR, Demonstracao: "n" parénteses “n" parénteses eee ofooorf)) | fe ebbrr yl) | ‘n" parénteses -[s[de(e*or")) j ~ “ne parénteses af a(acay")" | 22 1 Potenciagao Generalizaca Numa poténcia com expoentes em PG de poténcias, repete-se a base, ¢ expoente expressa-se como soma de uma PG. “p" parénteses ivmnpeNeacr Demonstracao: “p" parénteses - 7 eaeeee,rrn—_ | m m my" |... at [an(ar) } = at..[at(ar(ar-<™)] | = l \ parent p" parénteses "p" parénteses “p" parénteses 3 (14 mem? m9) = “p" parénteses Os Segredos da Algebra para IME/ITA/OLIMPIADAS, 23 Exemplo Resolvido 09 "10" parénteses are Determine [3 (3(3/) } . Resolugéio: Podemos escrever: "10" parénteses "40" parénteses . iO"parenteses _ 4) 2 2 2a 2 T | 2-(210- (2-@") } mb /[2..(9) } 3 PP) "10" parénteses: "10" parénteses = (2-(20?) P ge-coes) (2. (3137) J = 210 Exemplo Resolvido 10 parénteses ca [x 1{ or] Resolugao: Podemos escrever: "x" parénteses «loft al Exemplo Resolvido 11 x "2016" parénteses -\10 Determine 2.(8(2(e)")"| 24 1 Potenciagao Resolucdo: Podemos escrever: "2016" parénteses 10 (402016 + 1_49) ) (107 10 la(eeey") seat 10] 2017 T ) eet") Problemas Propostos Questdo 1.11 (Harvard-MIT-2012) 128) se 44° = , encontre o valor den. Questao 1.12 "2016" vezes ( 5 ey | Determine ( Pa y J \ Questao 1.13 _ — — Determine x” -x* -....x% Questao 1.14 a “x" vezes betermi (a(x x any) termine |x te (x . (x )) } . J Os Segredos da Algebra para IME/ITA/OLIMPIADAS Questéo 1.15 "60" vezes Determine a°b4 .a°b* Questio117 Determine x-(x2)'-(2) (x! Sugestdo: Use 1? + 27 +37 4...+n 2 _n(n+1)(2n+1) Questao 1.18 25 Determine x (2) fe) G4 P(e Sugestao: Use 1.2+2 343-44 enfant) = MOeMlnr2) Questéo 1.19 Determine 02) (oe) {wey Sugestdo: Use 1:2-3+2-3-4+...¢n-(n+4)-(n+2)= n(n+1)(n+2)(n+3) 4 26 1 Potenciagso Agora varnos ver ao bases fracionarias 6 08 Expoentes negativos, Vary 106 lay 4.3) Poténcia com Expoente Negativo: Numa poténcia com expoente negativo, inverte-se a base. . e-SE , @ colocs expoente positive 068-869 4m m_ {1 sam | a (2) 2a wo ymeN@aeR™ Consequéncias: ow (2)'= (2) = (2) "= (3 oy (2)"= (27 = (2) Exemplos Resolvidos 12 old ° Gy -G)-4 » (3) of Wee 1.4) Diviséo Composta: Numa diviséo composta, podemos alternar os 9s expoentes s de cada “termo 10 oo diviséo composta. Os Segredos da Algebra para IME/ITA/OLIMPIADAS. 27 Exemplos Resolvidos 13 ptigtiot.gtio g2.2 _2 4 3? i 9 je a i(2\] (a) 3 b) a att iy tatty tattsa3.p? = a a " b a [p } 7by (al) (a) Vimos as bases fracionarias. Agora veremos os expoentes fracionarios. Sigam- me! 1.5) Poténcia com Expoente Fracionario: Numa poténcia com expoente fracionario, 0 denominador € 0 indice do radical, @ onumerador é 0 expoente do radicando. a) svmeNeaeR,- b) (YaJ":vmneN, neOeaeR,, ¢) qd) e) 28 1 Potenciagao Exemplos Resolvidos 14 1 (3 a) 162) - 6 ~ 162) 24. i 5) (3) 3 (3) b) 64 thes = 164 =4{24) => 164) _ 4920 (2) (3) => 164? 16.4) =32 ja {act fet? pf la=a Problemas Propostos Questao 1.21 Questao 1.22 7 pM (oa% Simplifique a . Simplifique 2a"* asp (o's Questo 1.23 (AHSME-1954) a oval 130, {4) lest 2 valor de za-a + zs -|64 2 |-(32)3 é: 13 3 / 1 a) 1— |— Z wa aes d) 15 c)1 ds °) i6 Os Segredos da Algebra para IME/ITAOLIMPIADAS 29 Questao 1.24 (AHSME-1971) ~ f 1) 1\ zs se Be| sae = | 16 [2 8 ty ot) + +\ jr ‘oa 2 |-entdo S é igual a: ) ray art : oi |1-23| mite 32 1-222 21 } ) 1 1(, om 1 1}422 e)— os[78) 0! Questo 1.25 3 2 (32) (.-32 -3)7 : Reduzindo a expressao (-2) (-a %) {* }{a )[+! y } -obtemos: 1 a) a b)2 c)Q 7 d) Q e) a® Questao 1.26 Simplifique ;| ———————- 30 1 Potenciagéo Questao 1.27 3 b” Determine [= | : n Cc Questao 1.28 Questao 1.29 Sabendo que n é par, determine: Sabendo que n é par, determine: : a" a? Os Segredos da Algebra para IME/TA/OLIMPIADAS Y Capitulo 02 - Radiciagao Introdugao Atadiciagao nada mais 6 que a operagao inversa a potenciagao, ou seja, ela é& utilizada para representar, de maneira diferente, uma poténcia com expoente fracionario. Vamos conhecer essa ferramenta que nos ajudara a resolver varios problemas. 2.1) Definigao: Dado um numero real ndo negativo a e um numero natural m>1, chama-se raiz m-ésima de a o numero real nao negativo b, tal que b™ =a Ya b <> b™ Exemplos de Aplicagao: Exemplo 01: S27 -3 « 3-27. Exemplo 02: (2401=7 <> 74 =2401, Exemplo 03: 915625 -5 <> 5°= 15625. 2.2) Como Expoente Fracionario: A raiz m-ésima de um numero a poder ser definida como sendo uma poténcia de a, com expoente sendo o inverso de m, assim: Consequéncia: Se tiver expoente, esse fica como numerador. li Exemplo Resolvido 15: Efetue ‘9a. Resolucao: Podemos escrever: ‘Ya = a 32 2 Radicia¢ao g Exemplo Resolvido 16: Mostre que iq? - 121. 12 le] Resolugdo: Podemos escrever: Sy 4t2 44 8) 49? =121. Exemplo Resolvido 17: Mostre que 92 =2. Joti, Resolucao: Podemos escrever: 32 = fe 2 2.3) Propriedades da Radiciaca' ; ‘As propriedades da radiciagao facil e equagdes que envolvem raizes. aM os calculos de express6es numércy P14. Produto de Raizes de Mesmo indice: Num produto de raizes de mesmo indice, repete-se 0 radical e efetua-se 2 multiplicagao. Ya. ="Ya-b Consequéncias: a) ab Ye..=Vabc.. b) P.gd Exemplo Resolvido 18: Efetue Ya-¥b- Yc. Yd. Resolucao: 4a. 4p. Ye - Yd = “Yfabed. Exemplo Resolvido 19: Efetue 4/256. 481. Resolug&o: Podemos escrever: 4256 . YBi - 256-81 - Ya4 34 — 424 ~ 12. Exemplo Resolvido 20: Mostre que 4/a2 . 4{b3 . 4/c4 — o4/a? Resolucao: Podemos escrever. Os Segredos da Algebra Para IME/ITAJOLIMPIADAS 33 P2, Diviséo de Raizes de Mesmo indice: Numa divisdo de raizes de mesmo indice a » fepete-se o radical e efetua-: divisao. we Numa divisdo de raizes elevada a poténcia, rey cla Pete-se o radical e conserva-se a pol Exemplo Resolvido 21: Efetue 5 Ss Resolugdo: Podemos escrever: 5 rs 5 Exemplo Resolvido 22: Mostre que re Resolucdo: Podemos escrever: eer [2m yee a ~ ae 19520 32 2-3" 19 Exemplo Resolvido 23: Mostre que “ito 2.9 Yo Resolucao: Podemos escrever: 1920 32 2032 19f42 E- — 19f20-3 19g? = gf “ge ae =? 19) b= WF 1g, Qt 2. Radiciagao 34 —_———_——_— ~Problemas Propostos ~ ~ —_ ——— Questo 2.1 (CN-15 964) a ™~ 4 fa +a ~~ que Simplifique Ta Ya . va Questdo 2.2 (CN-2000) _ — Sabendo que Yx2 = 1999°, fy = 1999" e YF = 19998, com (x>0, a 1 z>0), ovalor de (xyz)3 € a) 199° b) 1999° 1 c) 19999 d) 1999 e) 1999°° Questo 2.3 (AHSME-1956) , 4 4 Simplificando ik Ya? 18 Yao , temos: a) a's b) al? c) aé 4) at e) a? Questo 2.4 (AHSME-1998) Se N>1, entéo INININ, vale: J a) N27 et 13 1 b) NO d) N27 e)N a c) NS Os Segredos da Algebra para IME/ITAIOLIMPIADAS Questao 2.5 (Harvard-MIT-1998) 35 Dado que re s sao inteiros positivos e primos relativos, tal que 2(v2 +Vi0) = —-,encontre res. 5(V3 + 5) Questdo 2.6 [ab pa-b ‘Simplifique a expresso 2-9 pa pe Questéo 2.7 27> pe, pe? aa Simplifique a expressaio 8-8 pie si . a .b? 4b? ab Questao 2.8 Simplifique a expresso Questao 2.9 ‘Simplifique a expressdo 36 P3, Raiz de uma Raiz: Para a raiz de uma raiz, repete- se 0 radicando e e multiplicam-s “Se Os j mn nia — Val; Consequéncias: monp a yy % - a Trt vezes sn yez0s mmm.-m » (Yee Re "m" vezes “m" vezes _ 22203 c) {ite Va -. wnnezes “n" "vezes m7 2.2.2.2 0 ee 22h e ’ » ae 28 Pie 88, h) Peg EH tig oe | {fefre - meet Ff Al. |e - 2. Radiciacgao indiogs vm, neNeaeR, "n* vezes (ive -a). Nie "a [__*n* vezes Pe ea fu PE pene. B oy Ya =B Ve Os Segredos da Algebra para IME/ITA/OLIMPIADAS 37 » Rm ee SE ah Exemplo Resolvido 24: Efetue {8m . Resolucdo: Podemos escrever: Wim =m => im = 24m. Exemplo Resolvido 25: Efetue 94/3/10 . Resolucdo: Podemos escrever: Wo -7 29770. ARN = 70 "2016" vezes 32018 Exemplo Resolvido 26: Mostre que “y ¥ Ya -=—Va. Resolugao: Podemos escrever: 2016" “2016" "vezes “ 2016" vozos "2016" vezes Whe - R333 e Wwe 3 % 6 "2010" vezes Exemplo Resolvido 27: Mostre que “Vyvva =*—Va. Radiciaca 38 2 a Gao Resolucao: Podemos escrever: "2010" Vez pro vezos "2010" vezes [fe -#226. Vile = "40" vezes 10] 510f 10 Exemplo Resolvido 28: Efetue 7 yf 7 Y 2p . Resolugao: Podemos escrever: “ol yezes "10" vezes y10 510, _| tof 710[ 510 2' 210-10 ={7) 77% oe = E25 100 > E=*2 a e176) Exemplo Resolvido 29: Efetue Resolugao: Podemos escrever: 2 2 23 52 ME eR Pp eG. m 4 43.2, Exemplo Resolvido 30: Mostre que ° a] 3Y va = Resolugao: Podemos escrever: e=% (m+2)1-m) 3.2 an Lashes 32.38.34 243444..0m ESV Ve = SS Sng . etittng 1 {2=mym-1)] (m+2\m—1) (m+2)'1—-m) a) [3 — 2 E="———_a = a3 - Bo. 31D. 30 Exemplo Resolvido 31: Mostre que 4 Be - YET Os Segredos da Algebra para IME/ITA/OLIMPIADAS 39 Resolugao: Podemos escrever: oo] Yq — BBM eg gi® ge. g30 oQgie 38 ,a0 530 e9(p5t 915-530 eRe A _ RSIG _MFTGIESM Exemplo Resolvido 32: Efetue 4) ¥/ a5 Resolucdo: Podemos escrever: sal ey iq c-Weep . - E Ee Ee Bp oe FEE Ep ee 5 8p | O] x. AEE A Exemplo Resolvido 33: Efetue Resolugao: Podemos escrever: fy nin c-We _ ER el) (3).(2).(1 2 Rew. SEE, SE, Exemplo Resolvido 34: Mostre que ER = x. Resolugao: Podemos escrever: fb? [Sa 2gQ10 29,15 20,16 co A[E Pp RA |g WEN, > 20/510 15 16 8 ee Be gato << K. E/E Es Exemplo Resolvido 35: Mostre que py = my 40 2 Radiciagao Resolugado: Podemos escrever: x2)' 29 (32) Xen 3 2)" tg ete | er, x Eg ee. el | Ge [be [ar Exemplo Resolvido 36: Mostre que “| Resolugao: Podemos escrever: _ — eo 5 pe = ee , 2] a) fay > Ba 7% Bay® * Problemas Propostos Questao 2.10 simpitique F499a Ja 4a a Ya Questao 2.11 Simplifique +3 Os Segredos da Algebra para IME/ITAOLIMPIADAS a Questao 2.12 == anyon Simplifique WwW a Questao 2.13 Vi implifique J—-=— > e-24fe2)" (0°) = EeS4 a *)-(p°) f & Ye? 465 = 295 b>. 2 Radiciagao 44 Exemplo Resolvido 39: Mostre que 5- 910 = 1250 . Resolucdo: Podemos escrever: 5.40 Ys? 10 = 5 F0~Vi25-10 = 5.30 = Ye, Exemplo Resolvido 40; Mostre que 72.1959 = "940.720. Resolugao: Podemos escrever: E-7.Y0 = e=ef72)" 10 > & = 19f72).49 LER 2018/52014 21 2 4° Exemplo Resolvido 41: Mostre que Resolucdo: Podemos escrever: _ pare [52074 —=201 7 Ee. aot 3 => swe E = 2016) Es B Exemplo Resolvido 42: Efetue er 3 Resolugao: Podemos escrever: 3 B_J3 . B_apre ero Se Problemas Propostos Questao 2.22 Simplifique “{—_\?_ va Ae oe , Questao 2.23 Simplifique 3210 S77. ai. Os Segredos da Algebra para IME/ITA/(OLIMPIADAS 45 P5. Raiz de Fracao Composta 01: A raiz de uma fragdo composta @ igual a cada termo da frag elevado a expoentes alternados. Jo composta 2 Radiciagado 46 Os Segredos da Algebra para IME/ITAOLIMPIADAS 47 Exemplo Resolvido 46: Mostre que Resolugao: Podemos escrever: Pe. P6. Raiz de Fracdo Composta 02: A frag fragdo composta das raizes € igual ao radicando elevado a expoente fracionarios @ alternados com os denominadores sendo 0 produto dos indices de acordo com as raizes que os comportam Re Ne eae —Ne_| / ; 9 _ {tans} om), (Ya) vmnpen eabceR, 8 2. Radiciagao Consequéncias: Os Segredos da Algebra para IME/ITA/OLIMPIADAS 49 ~ ate (4) (4 Exemplo Resolvido 47: Mostre que 1 : Resolugao: Podemos escrever: Exemplo Resolvido 48: Efetue Resolucdo: Podemos escrever: 5.5.5 acca £55 Fe0 fee? —_ (280) (425) (25 Wa fyias caa® 4 es 7s E=- (250) fae) (a > E=2118) 4 =) 43) E=22.3°5.78 2 Radiciagao 50 4 { 4025 aos | |-zqae) 79 TeIe 25 Tare ht _ ores al za), {73 a 25 F978 *z5-]) 400 3600_) (#28 25+ 400 604 (Gee sas HH SIS S168) _, Eg 49 6a =a (2) 800 | _ {ro | _, e-at?) Exemplo Resolvido 50: Mostre que Resolucdo: Podemos escrever: Problemas Propostos Questao 2.24 Questao 2.25 —— x Reduza 4 B Simplifique 8 Questao 2.26 Simplifique Os Segredos da Algebra para IME/ITA/OLIMPIADAS 51 Questao 2.27 simplifique {x2 x4 Vx? = Questao 2.28 (*)_ (2101.2 99) Determine 0 valor de n, tal que x . a) 102 b) 101 c) 100 d) 99 e) 98 a(a -1) ral (n—1)-° —n-grt +4] Sugestao: Use S = —__ a1 (a-1)" Para mais informagdes sobre essa soma, consulte um livro sobre progressdes € séries. Questo 2.29 —BLtadicals Sendo n um numero impar, simplifique _n(n+4 a)t bx 2 c) x" e)x z Radiciagao 52 eremos varias series finitas. Atengao aos racigg;, Hl A partir de agora V' a ig dao uma visdo além do alcance. Vejamos! i ni demonstragdes, Po! ling dey je Radicais: Sas séries finitas de radicais. Séo Series que ear ferramentas rapidas e eficazes na hora da prom ~ 2.4) Series Finitas d Nesta segdo, veremo: muitos calculos e S40 0 de Raizes na Forma 2”. $1. Multiplicaga a Esta primeira série aborda produto de radicais cujas poténcias aumentam o> IM em PG, o resultado é a raiz de indice da poténcia —_—_, € 0 radicando € a so uma PG. Ma de le - m" vezes “im vezes 2 [oma a?” | ymeN Demonstracao: "m" vezes E=Vavava => E= ya a-a2 > e-Weer a oeiibee™ oe. rr | = a eli er “m" vezes reer ee [i242 ey D4 “m" vezes gm rte he “m" vezes " Sen ". a aan a® — e- a > E= Wa 2) 2R2 [a [nae => E= Os Segredos da Algebra para IME/ITAOLIMPIADAS 53 Exemplo Resolvido 51: Mostre que Ri Resolugao: Podemos escrever 10" vezes mB. " vezes Exemplo Resolvido 52: Mostre que hevievieJie = Resolucdo: Podemos escrever: "25" vezes (z 2 24, 225 = | 2 ‘| hefieie > €- he 3 e-1e ) E= (242% ) 2 2 1) "25" vezes as) 5 “52923 ( 2 > E=2 } a> E-2 2 ievievie = 2 5 S2, Multiplicagao de Raizes na Forma m’. Esta série é uma generalizagao da série anterior, o resultado segue o mesmo raciocinio. jwmneNeaeR, Demonstraco: _inivezes _intvezes ane veres aly EWU se qe oe Wa “n" vezes cn" vezes — i veres Wgygi-mem? memes | remem sce? E= ry faitm+m? + >= E=- ims: 2. Radiciagao 54 Demonstracao: mandeies "n" vezes E-- "ar ar War => Ba Man nf Yan qnm "n* vezes ees 2a amit 2 mf maf n-(temam? sa =e VY Yaar oe gn a { ) “ar veres fan tem+m?.am® 1) fa Tel 4 {m0 14 mle] ae —— sot mi (m1) | mm mh (moa ouE=a_ JouE=a (m9) "5" vezes —— Exemplo Resolvido 53: Efetue %s a5 We Resolucdo: Podemos escrever: "16" vezes poets Exemplo Resolvido 84: Efetue a 1 OF Resolugdo: Podemos escrever: "16" vezes 4 104 ee to! (* "] 1o!6| (1 E=°97 197107 EL 14 et) see V7 Os Segredos da Algebra para IME/ITA/OLIMPIADAS 55 "46" vezes (1015-1) a f | 9-108 sof 197 TF _ 7.91086} 3, Divisdo de Raizes na Forma 2”. Esta série € com divis&o, em vez de produto, segue 0 mesmo raciocinio das séries anteriores. Demonstracao: "m+" vezes "mst mt vezes _ " vezes E=a-\a:Ja > E= la Wawa SES "m+1" vezes ‘mst vezes —ymet ay ) ge mtr par oom 3 a § a 3 7 N ; se m for impar. "met" vezes Demonstracao: "mst" vezes "mt" vezes le lev 3 ener Warea™ —e—_— — ES 2 Radiciagao "mst" vezes = yezes_ — 1-244. 4(- [| (27) ee Va (7) CEFF vezes mt Exemplo Resolvido 55: 3, ay +3 =a\ Resolugdo: Podemos escrever: "9" vezes E- (3: ¥3+/3 > E= "40" vezes (= 2 } Exemplo Resolvido 56: (5+ 5+ V5 = Resolugao: Podemos escrever: 9) 0 12) e-\o: 45-05 > e- Vs 3 se= Vs? «E-5 $4. Diviséo de Raizes na Forma m". o mes Be EVBAO Ge RAR S INA OME Te Essa série 6 uma generalizacdo da série anterior, 0 resultado segue raciocinio. ;¥mneNeacRy Os Segredos da Algebra para IME/ITA/OLIMPIADAS Demonstracao: "net" vezes aad vezes “nif” vezes, = E eV £- Ya Ware” oe Vasa” sa "n+l" vezes — _"net" vezes Inf +-mem2_..4(-mi" mmm 2 a [yhmem (-m) ate? 7 "net" ( ) (-m) 3) net vezes maf (emp (-m)-1} = — mat "net" vezes la: Ya: Va / ; sen for par | _int vezes it (==) 7 mt (Ya+ E} ; sen for impar "n+1” vezes, Demonstracao: ree yess _tnettvezes es arcana. apm nm? qtr oh) 57 58 2 Radiciagao “n+1" vezes ee Wan "Yan fa" = “net vezes — ee a” = Lea = "2n- " vezes ( 4-102" ant att toe't) Exemplo Resolvido 57: Mostre que . Resolugdo: Podemos escrever: wnat 7 on) 2n41" vezes toni [| (i: 10") 10% | E- 1G. Ya +E - ) a | 4 1-102" "2n41" vezes - Fare] mae _ le) = E-al 19f 1062 19g =a 110°") "2n" vezes (sem 1 ido 58: Woe _ | | Exemplo Resolvido 58: Mostre que Gx Se Ix =x. 7-67 Resolugao: Podemos escrever: Os Segredos da Algebra para IME/ITA/OLIMPIADAS Questao 2.31 ABE ABE ABE BF 7 Qual o valor de E Questao 2.32 Qual o valor de E = 'Y2 B.Yo?.1YD®.1Yo7.__.. 19909 > Questdo 2.33 Qual o valor de ve Bi 4 22 Questdo 2.34 Efetue §]7)7 47 7v7 . Questdo 2.35 Se x #0 enéumnumero par, determine - wey 2 ax 3 bx 2 41-2") ax 3 e) x12" Questao 2.36 — 2 Radiciagao 60 Questao 237 4] 3 4.3 43 43 = Qual o valor de te x? qx de. Me 2 "n" radicais Questao 2.38 656.55, 59,5. 65 Qual o valor de OO YX YO VK ? “n” radicais Questdo 2.39 10} 1 Qual o valor de i x? q x? 192. NE 2 "n" radicais Questao 2.40 2 Simplifique x" 1 Questao 2.41 x Qual o valor de “n" radicais Questao 2.42 _ Determine o valor de defen? de Gar ay xi 2h27 b) x(rt}2r o) ha" Dica: Soma dos termos de uma P.A.G- 4 x2" _afa=1) (n-th 0 4] a ° -1) Para mais inf lormagées € séries, es sobre essa soma, consulte um livro sobre progress Os Segredos da Algebra para IME/ITA/OLIMPIADAS A partir de agora, veremos as séries infinitas. Atengdo aos raciocinios que aumentam a visdo do leitor. Venha comigo! 2.5) Séries Infinitas: Nesta se¢ao, veremos as séries infinitas de radicais, Cujos raciocinios fornecem uma boa maturidade para quem os estuda. So séries que facilitam muitos calculos e so ferramentas rapidas e eficazes na hora da prova, vejamos. $1. Radicais em Soma: ‘Aqui veremos as séries em soma, acompanhe cada caso com atengao. a) Soma de Radicais Simples: ja+ja+Ja+ Demonstracao: x-\aryatdare = x2=atfa+Jarn = x? =arx = -x-a=0; A=(-1)?-4.1(-a) > a=1+4a; xe Ne = y_tivivda 24 2 “ a+ a+ Jaz..0 = Livia a :a>d. b) Soma de Radicais com Termo Fora da Raiz: 2 Va+bJa+bJas on - PEM +4) Se ;a>0eb>0 Demonstracao: x=\bibJarbla = Vat. = x2=a+bja+bVat...n > x? =a+bx = -bx-a=0; = A=(—b)?—4.1.(-a) -. A=b? 44a; xe rob) + vb? 44a x b+ vb? 44a See aa 8, x = Db + 4a 24 z ;a>O0eb>0. 62 2. Radiciagao c) Soma de Radicais com Termos em Produto: e] jyab + byab « bab. = be vb" +4ab| oe Demonstracao: —————— — x= ab + byab + bab + = => x2 x2-ab+bx => x? -bx ab =0; = A=(-b)-4.1.(-ab) = A=b? +4ab b)tvb?+4ab ic — DaNb? + dab . 4 - 2 “ = 2 ib ab +: Bp bab - 22D : d) Soma com Termos em Produto e um Termo Fora da Raiz: + Vabe 2 Vabe R’. 2 —=>= vb? +4 : lac + byac + bYac +...20 we ; vaceR, Demonstracao: x= yac x? =ac+bx = x?-bx-ac=0; > A=(-b)* -4.1.(-ac) > A=b*+4ac Io 2 ~(—b)+ vb? +4ac x= DEwb? +4a0 4 ered i 24 ° 2 2 2 . ac = bJac+bjac+..0 = Dt¥0_ +480 aE vace R’ e) Soma com Produto de Termos Consecutivos: La(a~1)+ Jata+ty+ Jafar tyes -ardf, vatarte®- Demonstracao: x= Ja(a+1)+Ja(a+1)+ falartj+...0 Os Segredos da Algebra para IME/ITAOLIMPIADAS 63 wp 2 =a(a+t)+ falas t)+ fa(a+t)+...2 oy x2 -a(art)+x = x? -x-a(a+t)=0; 3, a=(-1 -4.41.[-a(a+t)] > a=1+4a?+4a A= (1+2a)*; “ x=tta 4)+ # (142a)° 441428 2+2a => X= Ke 2a 2 2 fala 1)+ Jfa(a+1)+ Ja(a+t)e.. f) Soma com Produto de Trés Termos Consecutivos: [ae fata=1(a~2) += _teyiedata a2) Demonstracao: x=,fa(a+t)(a+2)+ Ja(a+1)(a+2)+ fala+1)(a+2)+...0 > x =a(a+t)(a+2)+ Ja(a+1)(a+2)+ fa(a+1)(a+2)+. ; va(at+teR). = x? =a(a+t(a+2)+x > x? -x-a(at1)(a+2)=0; > a=(-1f -4.1. [-a(a+1)(a+2)] » A=1+4a(a+1)(a+2) ; ~(-1)4 yt+ aie 1+ f1+4a(a+1)(a+2) {a on «Kei SS aS (a+1)(a+2) ja(a+1)(a+2)+ \alast(a+2)+...2 : 9) Soma de Termos Consecutivos e com Termo Fora da Raiz. eae a(a +1) +(a-2),fa(a+1)+(a-2)Jala+t)+—2 eas v4 *=Va(a+1)+(a—2)fa(a +1) +(a—2)fa(a+t)+ © 2 Radiciagao 64 =, ee a(at ty+(a-2)yaa +2 2) fala A) roe = x2 =a(att)+(a- 2)x = x°-(a-2)x-a(a+1)=0; = a-[-(a- 2 —44[-a(a+t)] = &=(a~ 2)? bdalas +1) = azar ad +4+4a 2448 2 AR 5a2 +4; -[-(a ae +4 Test 2) be va 2 ' Soma interessante: h) aiza+t) (a+) fa(2a+1)+ (a+) fa(2a+t) +07 = 2a+4 Demonstragao: x =,Ja(2a+1)+(a+1) fa(2a + 1)+(a+t)ya(zas tyr = x2 =a(2a+1)+(a+1)Ja(2a+1)+ (a+ 1) Ya(2a +t) +. => x? =a(2a41)+(a+1)x > x? -(a+1)x-a(2a+1)=0; > a-[-(a+t)} -4.1[-a(2a+1)| = A=a242a41+8a" +48 va(att).(a~ a-2)e a. = A=9a246a41-. A=(3a+1)"; [ies [elevate x- aise : feezacd. i) Soma com Inverso do Produto Consecutivo: la(a—1)+ fata 1) fala tre al; va(a-1e B+ Demonstraca x= ata) a(a—1)+ Ja(a tr Os Segredos da Algebra para IME/ITA/OLIMPIADAS 65 = x? =ala-1)+ fa(a—1)- ala) = x2 =a(a-1)+x = x?-x-a(a—1)=0; = aq(-1-4.4[-a(a-1)] > a=144a?—4a » a= (20-1); ss x —1t2a 1 5 x= 28 3 Zt la(a-1)+ ja(a—1)+ Ja(a—1)+ j) Soma com Termos em PA: ja? bla? + (ab) Ja? +(24 +d) Ve = a4] Demonstra¢ao: arb=ylaib)’ > avb= Va? +2ab+b? > ab= qe? sbidast) tarb=y[2a+by = 2arb= fai (arb): = 2a+b= fa? +2a(a+b)+ (a+b)? “*3a+b=(3a+b)? > 3a+b = yfa+(2a+b)] = a+b ~ fh? + 2a(2a +b) (2a+b) = Barb = fa? «(2a +b)(2a+2a+b) = dash = Fs arbyaasb)] ~ Logo, temos: atb= fe? sb(2asb) = arb— fa? + ba? +(a+b)(3a+b) © a+b= a? bila? + (a+b),fa? +(2a+b)(4a+b) 2. Radiciagao 66 Vejamos as aplicagdes desses radicais! Exemplo Resolvido 59: Efetue V24+V24...0. Resolucao: Podemos escrever: tafna2 ta v08 = Yai ore SE-B a e- = et? . (ae Nbr =2. Exemplo Resolvido 60: Qual o valor de la+3a+aVa- Resolugao: Podemos escrever: 3+Vee4d _ p_3+verT6 E=V4+3V4+...0 > E= 3 - =e 3tv 348 8 ong 2 2 2 Exemplo Resolvido 61: Qual o valor de i124 4fi2+..0 =6? Resolucao: Podemos escrever: E=Vi2+4Ji2+..0 = E=3-444)8-44...0 4+ V4? 44.3.4 4+ V16+48 4+ eo 5 pe s E=- 2 2 2 448 zeit. e-2 s \12+4J12+...0 =6 Exemplo Resolvido 62: Efetue /3.8+5/3-6+...0 . Resolugao: Podemos escrever: £=\8-8+58-8) 5 oe SHV +4-3-8 2 sen Stet, e-5it, 516 3 b a >eE 51 12578 - 2 (Os Segredos da Algebra para IME/ITAOLIMPIADAS 67 Exemplo Resolvido 63: Qual o valor de faz + fa2 +J42+...0 ? Resolucao: Podemos escrever: en ya2+Va2-Ja2= eS E- [67+ 67+ B Tow 6+1 6+1 6+1 = E=6+1 -. ¥42+ 42+ J42+..@ =7. Exemplo Resolvido 64: Efetue /2-2.3+J2-2-3+...0. Resolugao: Podemos escrever: e-\223-V2 237.0% e = f2(1+1)(1+2)+ 2+ 1)(1+2)+..% 4+¥i 4223 5 elvis 5 p-149 u ee 2 2 2 oe ett = e-3 « V22-34\2 235m 4, Exemplo Resolvido 65: Efetue 72 +6/72+6V72+6...0 . Resolugao: Podemos escrever: e-\72-6(72-6f72-8.= = E-8-9+6\8.9+6)8.0-6. - 8-2)+ 5-82 44 = E-jee- (8 Te _(B=2) 5-844 ble) (6-2) (E-s. 2 E ; 6+ 320+4 6 +324 = ¢- 8418 | Ela. = —-—eS Ss Ef 2 2 2 Exemplo Resolvido 66: Efetue 210+ 114210+11Jo Resolucao: Podemos escrever: &=4210-1Wato Woe = 10(21)+11,10(2-10+1)+11V...20 pee E=10(2.10+1)+(10+1) ffo(2-10-1)+..0 = 2-10+1.. [Ecat 2 Radicia¢gao 68 Exemplo Resolvido 67: Qual o valor de y12-11+V12-114 9 Resolugao: Podemos escrever: fini VIR TT = ft2- (12-1) + ff2- (12-1) + => 12-114 V2 41+...0 =12. Exemplo Resolvido 68: Qual o valor de y9+7y9+10V9+13V os 2 Resolugao: Podemos escrever: E = 9+ 7y9+10V9+ 13¥...00 => E~ 3? +73? +(3+7) (82 +(2-3+7)V.0 = E=3+7 ¥9+79+10V9+13¥...0 =10. Problemas Propostos Questo 2.43 (AHSME-1954/Stanford-2010) Se x= y1+ Hida vie , entao: a)x=1 b) O2, mas finito Questao 2.44 (Harvard-MIT-2000) Qual 0 valor de vi +2414 3y1 +4y1+5JT4+e= ? Questdo 2.45 (Harvard-MIT. ee stao 2.45 (F EE Qual o valor de Vie +3y16+7 16 +1116 + 15V16 +-- ? Os Segredos da Algebra para IME/ITAYOLIMPIADAS. 69 questéo 246 _ qualo valor de ¥9+2 lo. sya. e4a.1Wer 7 Questao 2.47 qual o valor de 16+ 16+ 5¥16+9y9+13J16 +... 2 Questdo 2.48 Qual o valor de ya + 2ya+ 2a. 2 Questdo 2.49 Qual o valor de 24+ 224-4224 +07 Questo 2.50 Qualo valor de 42070 +2070 +/2070+... ? Questao 2.51 Qual o valor de faorsaps xyfatxen)e(nvae +(X +n) fa(x-+2n)+(n4.ay? +(x+2n)... ? Questio 2.52 Qual o valor de Jeo + 60+ Y60+... 2 7 2. Radiciagao $2. Radicais ¢ em Diferenca: x-ya-ya-Va-..8 ane la. => x2 =a-y = y2.x-a=0; > A=P-4.1.(-a) > A=1+4a; -1+Jie4a ee TO 2 la—Ja-va- ay eee Wat) vaeR, b) Diferenga de Termos Consecutivos: yala+t)- (a+1) fa(a+1)-...00 =a}; va(a+1)eR, Demonstraca x= Ja(a~t)-fa(a+1)- fala=1)- 0 =x? =a(a+1)-fa(a+1)— Jala+1) = =a(att)-x > x? 4+x-a(a+1)=0; = a=?-4.1[-a(a+t)] = Aat+4a2 44a 2 a =(142a): + 2 xe let 2ayt x citt+2a 2a Tan OR SxS Mae. la(a+t) - fata +1)- fala-1)-...0 =al: va(a+teR)- c) Diterencaido do Inverso de Termos Consecutivos: eed Os Segredos da Algebra para IME/ITA/OLIMPIADAS Demonstragao: xe fa(a-1)- ala 1)— Ja(a—1)- Joe = xX =a(a- 1)-yata- 1)- Ja(a-1)- J = x2 =a(a—1)-x => x2 +x~-a(a-1)=0; = A=? -4.1[-a(a-1)] > A=144a? 4a A=(2a-17; 2 1+ f(2a-1) -1+2a-1 2a-2 - => xootteant = 224 5 xea x oa : > xe SS nat. a(a1)- Java ~1)-fa(a-1)-f...0 =a-1); va(a-the RY d) Diferenga interessante: [fava 1)-(a+1)y2a(a—1)—(a+1)/2a(a-1)-(a+1)Vi..we =a-1 Demonstracao: X= yea(a~1)—(a+1),J2a(a—1)—(a +1) f2a(a—1)-(a+ 1). > % = 2a(a-1)-(a+1),fea(a—1)-(a+1) feala -1)-(a+t)vwe 2 © Xs 2a(a—1)—(a41)x = x? +(a+1)x-2a(a-1)=0; ~ 8=(a44? 4.4 [-2a(a—1)] = A=a?+2a+1+8a? ~Ba _(a+1)+ feat? > =Qa2 8=9a? art. A=(3a-1)"; => x= 21 3 xs Aste 3a-1 2a-2 2 > Xa 2 x=sact. ~(as 1) f2a(a—1)-(a=1)¥i.» =a-1 ; va(a-t)eR). 71 2 2 Radiciagao Exemplo Resolvido 69: Qual o valor de v20 V20—..00? Resolugao: Podemos escrever: bo- (20 Te 20-20 re => \20-J20-. 8 > §20-J20—..0 ts = 20 ~ /20- +0 = Exemplo Resolvido 70: Efetue v5 6-5-6-J5-6—..0. Resolugao: Podemos escrever: 5-6 -V5-6-...5 = |5-(5+1)- /5-(5+1)- 55+ 1)-...m . (5-6-y5-6-J5-6-..0 =5. Exemplo Resolvido 71: Qual o valor de vi 00-99- y100.99-f..= ? Resolugao: Podemos escrever: vi00.99- 100-99 - = Jw = f100(100-1)- ,f100(100-1)- = => ¥20-J20—...0 = 4, 100 .99-/100.99-¥...0 =100-1 > = y100-99-./100-99-¥V...0 =99. Exemplo Resolvido 72: Efetue 12-4124. Resolucao: Podemos escrever: i2-4 hea = 2.3-2-4)2-3.2-4fam => Vi2-ai2-a = 2.3.3 -1)- (3+ )y2-3-(3-1)- (B41) => Vi2-4 N2-4f w =3-1 => yi2-4 N24. =2. Os Segredos da Algebra para IME/ITA/OLIMPIADAS 73 s3. Radicais Alternados Radicais Alternados Simples: a+ Jae Demonstracao: Nesta demonstragao usaremos uma técnica de fatoragao que sera mostrada no capitulo sobre fatoragao — x2\a-VatJa—..0 = x? =a-Jarx => Jasx-a—x2 2 = atx=(a-x?) => a+x =a? —2ax? +x! = x4 -2ax? +a?-x-a=0 Somando e subtraindo x3, x? eax , temos: +a*-x-a=0 +a? -x-a+x? x? —x? 4ax-ax=0 > 0? (x? ~a— x) + x(x? xa) a(x? -a-x)+(x?-x-a)=0 = (?-a-x)(x?+x-a41}=0 « x? -x-a=0 (nao serve) oux? +x-a41=0. ?—4.1.(-a+1) . A=4a-3; Pex-at1-0 5A x= 2+ V4a-3 24 ol = va» Ba (a+ 1)~Ja(a+1)+Ja(a+t)- Vv yaar Semonstracao: *syala +1) Ja(a+1)+ fata +1)—¥o.0 s “afar 1) (at t+x => Tare =a(ast)-™ "a 2. Radiciagao > a(ar rx (ala +1) ey) > a(artrx ja(ava)f 2a(a+1)x2+x4 > x4 2a(as tx? x a(avt)+[a(a+)P =0 Fazendo k - a(a +1), temos uma equagao do 2° grau em fungao de k ¢) xt 2a(a+t)x? -x-a(a+1)+[a(a+ af 0 > x4 kx? —x-k +k? =0 => k? (2? 4 1)k (x4 x) <0 > A=| (2x? +)f 4.4.(x4 x) <> A= ax 44x? 41 ax ax > N= 4x? Fax 41 o> A= (2KH17P k | (2x? +a) fs face? os PR ERK 24 2 2x2 444244 2x2 4.4-(2x 41) => ky=-——~——._ ou: ky S ——_ 4 2 2 5 kya xP 4x41 OU ky =x? =x. (*) Observacao: E muito interessante essa técnica de encontrar aS raizes & uma equacao, em fungdo do parametro. Assim, temos: 18k? kK? =O > 4:/ k(x? +x +4) |] k(x? x) ]=0 = [a(asa)- (x24 x+1)]-[a(a+t)-(x2-x)]=0 » a(att)-(x? 4x41)=0 ou a(a+t)-(x?-x)=0 xP 4x 41-a(a41)=0 ou x?-x-a(a+1)=0 (nao serve) x axtt-a(ast) > x24x41-a2 a = b= -4.1-(1-2? -a) 3 hated da? 4a Os Segredos da Algebra para IME/ITA/OLIMPIADAS 75 44 Jaa? +4a—3 x -1+ Jaa? -4a—3 a = Nes tea 5 2 2 Log SSS => | ata=1)—Jata+t) Ja(ast)—Voe == Exemplo Resolvido 73: Mostre que Natya 19 a Resolugao: Note que 7 >1, entao: Exemplo Resolvido 74: Mostre que 3 —3+V3- Resolucado: Note que 3 > 1, entao: —= 4+ 4-3-3 14+ 12-3 Babe ow = 6-8 = ee > eae = e-lt8 5 e-2 = \b-vB+8—..@ =1. 2 Exemplo Resolvido 75: Efetue [8-9 y8-9+ ¥8.9- Resolugao: Podemos escrever: 2 144.87 +4-8-3 F=Y8(8+1)- fe.(8+4)+ fo-(8+1) Fe = B= SB NRTSETS fp a1 P85) 2 ~ 2 2 Radiciagéo _ ‘oblemas Propostos. ~~ Problemas Propostos _ = _ . Questao 2.53 oS Questo = — 21 at a 2 \ 6 Vie Vie questao 2.54 (RussiallMO-Longlist-1969) Pro orre a identidade 2 Prove que, para a> b* , oct o-olarde tye = p22. Questdo 2.55 Qual o valor de y60-760 —7¥ 60 -7v...0 ? Questao 2.56 Qual o valor de 1762-11162 —1 4162-1 4/162 0 ? estado 2.57 Qual o valor de /2450 - 2450 - 2450 — 42450... ? Questao 2.58 Se Qual o valor de Ver + W= 00? Questo 2.69 _ — Qual ovalorde ,|22_ 39, [89 4 4 + Pema 2 Os Segredos da Algebra para IME/ITA/OLIMPIADAS. 7 $4, Radicais em Produto: Va vada ~ =al;a>0O Demonstragao: xo \alalan= > @=ax - x=0 ou x=a «. [\ayava.m =al: a>0 a) Generalizacao do Produto Anterior, para Raiz de indice m: mo my m1 x la'VaVa...x ="—Va|; vyme R-{0,1} ea>0 Demonstracao: xa WalaYane = x" =ax > x x= Va». MaVata = =" Val: vmeR {0,1} ca>0 Qutra forma: ; vmeR-{0,1} ea>0. vmeR-{0,1} ea>0. 2. Radiciagao 78 Demonstra¢ao: m n Mm _ an x afin ahr Mar => x=Var-x = x™=a"-x = X vme R-{0,1} ea>0. Exemplo Resolvido 76: Efetue 4100,100./100...2 Resolugdo: E imediato que vi 00,/100J100...2 =100. Exemplo Resolvido 77: Efetue Yosee 0 = ¥2. Resolucao: Podemos escrever: Weietans - 2p = sosoee se 4p. Os Segredos da Algebra para IME/ITA/OLIMPIADAS 79 a 3 ee { Bt 1 exemplo Resollvido 78: Mostre que | 8 ( 8(8...)8 Fl - Resolucao: Podemos escrever: 1 8 1) { Ne “ge J, o[ote..e al (8 .0)9 5 | \ } 5. Radicais em Divisao a) Radicais ‘Simples: Demonstracao: x-ya+VazVaz.0 => x? =arx > x? 2 8-a x Qutra forma: 2 Radiciag&o 80 Demonstracdo: ' / 1) on, 1 \m | x=jas/ae (ate )m | | : } ( ;¥meR-{0.1} ea>0. ; vme R-{0,1} ea>0. Os Segredos da Algebra para IME/ITA/OLIMPIADAS. Demonstracao: = Ta]; vme R-{0,1} ea>0 2 Radiciaga 82 eae e) Divisao Composta 02: Ez, :ymeR-{0,1 caso Exemplo Resolvido 79: Efetue 27 +27 + §27 +...0 Resolugdo: Podemos escrever: a \o7 +27 +27 =. =927 = 27 + 27 = V27+...0 =3. Exemplo Resolvido 80: Efetue 1024 + 4024+... . Resolugao: Podemos escrever: hoa. Giooa sa - 2 ond = {fi024 = load =... = V1024 = Yfi02a Pi0ad= mw = 82 = froze +fio2d+...0 =4- . 1 10 Exemplo Resolvido 81: Mostre que tto[11(11s..0)5 A Os Segredos da Algebra para IME/ITA/OLIMPIADAS Resolugdo: Podemos escrever: 1 10 1 = 441041 1 jr (14 ve)0 \° 1 fi 1 \10 1 (4) > [" frrcate re) aati), { Exemplo Resolvido 82: Mostre que 83 2 Radiciagao Exemplo Resolvido 83: Qual o valor de Resolucao: Podemos escrever- $6. Radicais em Cadeia a) ~ Poténcia em cadeia infinita 01: se x” ‘ =a, entdo [x= 9%]. Demonstracao: Demonstracdo: nas (SJ =a = Ve=% > x-a.. ¢) Poténcia em cadeia infinita 03: Demonstracao: 6 yf > = 1 1 ja = — 1 —_ ¥ =x MP OGL, gb ae" Os Segredos da Algebra para IME/ITA/OLIMPIADAS Por comparagao, temos 85 , ; » -te-% a xna., am al f_pas bi=b a Observacdo: Se xa, nao poderiamos usar 0 resultado. d) Poténcia em cadeia infinita 04: 1 Se x =a, entdo |x Demonstracao: a x ca > x —a [xea e) Poténcia em cadeia infinita 05: Se [xe Demonstragéo: he IK xv Xa - [x=a®). f) Poténcia em cadeia infinit Demonstracao: entaéo 86 2 Radiciagao Fazendo Ya-y => a=y", temos: = x’ -a > x! =y* = x=y (Por comparagao) Observagao: Para x = 2e x = 4, temos duas solug6es, visto que: 24 42 e 42-24, Logo x = 2} = 2" -a> 2a Fazendo Ja=y => a=y7, lemos: = 2-a > 2 =y? = y=2 ouy=4 (Por comparagao) = a-2 .. fa=4joua=24 ». Bate). ro = ak saa 4% a. x= 4) Fazendo 4fa=y > a= y‘, temos: = 4¥-a = 4% =y* = y-2 ouy=4 (Por comparagao) ua-44 .. fa=256). Vejamos as aplicagdes sobre isso! = a4. x Exemplo Resolvido 84: Se x” = 5, qual o valor de x? Resolugao: Podemos escrever: x =5 = x°=5 => x=%. Exemplo Resolvido 85: Qual o valor de ye* ? Os Segredos da Algebra para IME/TAJOLIMPIADAS 87 weve Resolucao: E imediato que va pe" Exemplo Resolvido 86: Qual o valor de YW 2 pe Resolugao: E imediato que 9/7 =7. ~~ Problemas Propostos tata ta 7 x4 ft s Gxt et > Qual o valor de E = “— Questao 2.60 Questao 2.61 | [ 2 261.9 29, 7,9 28, oo Qual o valor de E = ? [3 15] 19).9 191.9 16,9 fe. Questo 2.62 Questo 2.63 = Simplifique Simplifique Se a 2. Radiciagao 88 . Questao 2.64 _ Ques es, x me Qual o valor de x. sabendo que x*" = 2017? Questo 2.65 Qual o valor de x, sabendo que x“ = 27 ? Questo 2.66 = 1955 — 0 Qual o valor de 190 ? Questao 2.67 243585 9243 * 2 24: Qual o valor de 249/243 Questao 2.68 se wi =k emes a)1 b)3 6)5 ly } determine x. d)7 e)9 3 Questao 2.69 wet Qual 0 valor de 2/2 ? Questo 2.70 55) : Qual o valor de ay ? Questao 2.74 __ Qual o valor de * x, sabendo que x¥* = 625? Os Segredos da Algebra para IME/ITA/OLIMPIADAS 89 auestto 2.72 Qual o valor de x, sabendo que x Questao 2.73 ; te 19)” { { Qual o valor de x, sabendo que x '* =7? Questao 2.74 { i Qual o valor de x, sabendo que x'* =327 Questao 2.75 7 7 V7 Qual o valor de 7 Questao 2.76 41 1, ik} Qual o valor de 13 1" 2 90 2 Radiciacgado Desse ponto em diante, veremos as operacées com radicais, Parae e subtragdo, usaremos a estratégia de “colocar em evidénc;, efetuaremos a operagao. Para efetuar multiplicagao e divisdo, uy: propriedades de multiplicagao e divisao vistas anteriormente. Vejamos a seguir como operar com radicais! fetuay Son, a” © ent, SAremog 2.6) Operagdes com Radicais Nesta segdo, iremos dividir em duas partes: a parte “a” sera a adigig >~ subtragao, e a parte “b" sera a multiplicagéo e a divisao. Para efet s operagdes com radicais da parte “a’, devemos verificar se os radi semelhantes, isto é, se os radicais tem o mesmo indice. tuar as ICais sao a) Adicao e Subtragado: Podemos reduzir os radicais em uma soma ou diferenga, desde Que sejam radicais semelhantes. Exemplo Resolvido 87: Efetue 3¥4 +74 . Resolucao: Podemos escrever: 3N447V4 =(34+7)V4 = 3¥4+7V4 =10V4. Exemplo Resolvido 88: Efetue 209a + 8Y/a —149/a. Resolugao: Podemos escrever: 20Ya +8%a -149a =(20+8-14)%a = 20%a+8Ya -14Ya = 149. Exemplo Resolvido 89: Efetue mY/a* + pY/a* +qVa* Resolucao: Podemos escrever: mia" + pfa® ~ ifa* =(m+p+q)ia . Podemos também manipular os radicais de modo a ter termos semelhantes. vejamos alguns exemplos. Exemplo Resolvido 90: Efetue 8 + J33 . Resolugao: Podemos escrever: V8 +32 =J434 S69 VB + 32 = V4.2 + J6 JZ = V8+V82=2.244./2 = 8 + J32 = (2+4)/2 = V8 + V2 = 6,2, Os Segredos da Algebra para IME/ITA/OLIMPIADAS 91 Exemplo 91: Mostre que 2a [2704 « Yizea = (7 3a) 9a. Resolucao: Podemos escrever: ae Pite® + 05a ~ 2a a a ES = 2a Para + YBa 29ST Ae Sa => Ra- ahre* «25a = 29a -3 aa+5.4e =» 2a 2704 + Ba = (2-3a+5) Ye =, 2a tat + YBa = (7-3a) Ya b) Multiplicacdo e Divisdo: Para a multiplicagao e divisao, devemos considerar dois casos: se os radicais tiverem o mesmo indice e se os radicais tiverem indices diferentes. Caso 01: Se os radicais possuem o mesmo indice. Se os radicais possuem o mesmo indice, vocé vai utilizar a propriedade do produto de radicais de mesmo indice. Exemplo Resolvido 92: Efetue 2V2-5./11. Resolu¢do: Podemos escrever: 2y2-5V11 =10J2-41 => 2V2-5V11 = 10022. Exemplo Resolvido 93: Mostre que 3¥a-495 Ye? . 24d = 24. Yabo?d . Resolucdo: Podemos escrever: Wa-4Yb-Ye2 29g =3.4.2.Ya-b-c2-d = 3Ya-496 Ye? 24a - 24. Nabo2d . m Ya p Yb Exemplo Resolvido 94: Efetue Resolugaio: ma _m_fa pYb p Vb 2 Radiciagdo 92 10 ¥5 Exemplo Resolvido 95: Efetue 0 . Resolucao: Podemos escrever’ 1095 10 J5 _, 10¥5 A “as 4 V3” 4¥a 23 Caso 02: Se os radicais tiverem indices diferentes, devemos efetuar og seguintes passos: Passo 01: Tira-se o mmc dos indices, o mmc sera o novo indice, Passo 02: Efetua-se a divisdo do mmc por cada indice anterior, eo esultady multiplica-se pelo expoente do radicando. Passo 03: Efetua-se a operacgdo como no caso anterior. Exemplo Resolvido 96: Efetue Ya -Ub - 8 Resolugao: Efetuando os trés passos acima, temos: = Va-Y6 YE => & =A fare MR imp OB feran & = m08 fan yn (2° mp far b™.23mn Exemplo Resolvido 97: Efetue 29a 18a” . 25a . Resolucao; Podemos escrever: E=2.¥aYe0? 26a > B= 2.1%GF "Glr992)" 9258)? = => E=2 a* (2.3?) -a.25 — TERR Rett? 28.38 (52 F ge 2 a GT gh ge 1 Es 2a. 133 30.54 Os Segredos da Algebra para IME/ITA/OLIMPIADAS 93 Capitulo 03 - Racionalizacao Introducao Racionalizar uma fragdo consiste em eliminar o radical (ou os radicais) que estiverem no denominador, para tal tarefa usamos 0 chamado fator de racionalizagao. 3.1) Quocientes Notaveis Os quocientes notaveis sAo expressdes que vém diretamente dos produtos notaveis, ferramenta que vocé vera mais a frente. Seréo muito usados na racionalizagdo, nesta segao veremos os mais utilizados e a generalizagao, mais a frente veremos mais dessa ferramenta fortissima! 2 rs a) 2 asd. 3 3 p) SP a2 sab +b? a-b °c) =(a+b)(a? +b?). =at+a°p+a2b2 +ab?+b4. ~a®b +ab? -ab? +b*. af _pn 9) = alt gah 2p 4 ah 3p2 4b a hy ght _ at 2p 4 g3p2 —... bp", para n par. at —pn a i) SRP grt om 2p yah 3p2 ptt 20" para n impar . atb a+b _ jy ast atb Babe ath +b", para n impar. —p™1 - para n par . + 94 3 Racionalizagdo a ahd? amt al 2p gap? 4.4 pmt, 20" . ab a-b )) 3.2) Fator Racionalizante ee ee © fator racionalizante @ 0 fator que elimina o radical quando ee multiplicagao pela fragéo. Vejamos alguns casos em que usa racionalizante e outros bem interessantes. etuameg > MOS 0 fa, Caso 01: Quando o denominador é da forma VA : Note que nesse caso ao multiplicarmos o denominador por VA , fica: NN VA N NVA VA AVA Oe (vay Assim, VA 60 fator racionalizante. Exemplo Resolvido 98: Mostre que $ = 2. Resolucado: Podemos escrever: 112 1 1 See Pm ees ae. eRe” Bf ( By v2 2 Exemplo Resolvido 99: Mostre que os 70 . Resolugao: Podemos escrever: 1 _7 v0 7 7-10 7 7 eae =e EL, = N10 vio Ji0 io 10 (viay? “ Vio 10 Exemplo Resolvido 100: Mostre que Resolugao: Podemos escrever: W333 V2 ag 3V543 Va Joe 2 38.3 33 _ 336 aoa AST = B38 33 = S836 we te ag Viz 12 > x7 > B38 Vi2 12 ~ gd Os Segredos da Algebra para IME/TA/OLIMPIADAS 95 caso 02: Quando o denominador é da forma Yar Note que nesse caso ao mulliplicarmos o denominador por Yan ™ fica: yw RN NAR Assim, VA"™ 6 0 fator racionalizante - 3 Exemplo Resolvido 101: Mostre que —— = ‘xemp! q Ye Resolugao: Podemos escrever: a3 4209 3 3 eT 8 8 SP WYP Mpio-s Ys 19s Yr” HOS 1YD9 77 3 3.7 3 3.17 os igo 199310 > 3 1. 3 Exemplo Resolvido 102: Mostre que BABE p> 2 Re *solucao: Podemos escrever: Caso Neste s Quando o denominador é um quociente notavel *S0 Voce multiplica tudo pelo fator do quociente notavel EE OO — 96 3 Racionalizagay 12 Exemplo Resolvido 103: Mostre que nae - E-; Resolugao: Podemos escrever: 12 2 8 v5, _ 12 2 3-348) 313V5 3+3V5 3- 353+ 3N5 F- (38) 12 _ 36-36V5 36V5 12 ete —*) 34+3N5 * 3535 120 og gas 4/2 Exemplo Resolvido 104: Mostre que ve v2.(W7 3). V8 Resolugao: Podemos escrever: a2 _ 2 V7-VB ay2__ 4V2-(v7-J8) FB FB WB I (ah P 42 4V2.(V7- V8) av2__ 4v2-(V7-V3) > ™ _ ~~ Gh 7-3 a ‘ 4/2 Fp PF (7-8) Exemplo Resolvido 105: Mostre que ———— 3 a aio , R20 Resolucao: Podemos escrever: 0 3 Vie 3: fot F-2fio pare 7B Weaia ” Foase fe (of) = 3 7 ~ 210° s 3 sieht Vi-2fo oe 40 W720. ni 3 3. = 3 _3-W7+2/i9 ¥ Os Segredos da Algebra para IME/ITA'OLIMPIADAS 97 Sj a 1 ¥a9 Ya + Ya Io Resolvido 106: Mostre que ——__ - V** Exemp! Ff 5 Resolugado: Podemos escrever: 1 (f+? 8s (¥a)P Fe W- WR (ay +97 32+ (32) 1 Yao+Va+¥a ot Yao + 14 + Ya HE aay | We 72 1 Yao Yas Va i nn Exemplo 107: Mostre que) ————___—_____ - 3 - 9 981+ 54 + mm Resolucdo: Podemos escrever: 1 _ 1 Pee ai Fhe Fe hee 1 _ 1 B- = He, ° eae. Sea + fe (BY -( (88) 3% 1 B22 VAs Vas ee Yaa fie 382 1 fold ei. Yea. We « aa + Vie 8 3 Racionalizac&o Caso 04: Forme interessantes que Seguem a mesma linha de , Sting 1) Forma Be EF ¥ N _ vR+vB Ve KE RoE Va + JB -Ve rn N.(VA + vB - ve) > RB (K. P (vey N N.(JA + vB - JC) > Ieee JC A+2VAB+B-C N _N-(VA+ NB VC) a+B-c-2VA8 > Teie+ Je A+B-C+2VAB A+B-C-2VAB N N(R WB “6 ) (08-65) a+ VB + JC (A+B ay (2vaB) iON ON (VAs 8 YE) (A+ B-C-2VAB) RBC (A+B-C)? 408 EE A+ B+ VE ABE E Ye VRB E Ye VA-B- Je N N.(VA + B+ JC) RE aye N N-(Ji = 8 + JE) ~ RB A+2¥AB+B-C N ~ TROBE N ~ Tae ~ N(VA~VB+VE) 9.3 c_2/AB A+B-C+2JAB A+B-C_2VAB N-(VA + VB + VE)-(A+8-c-2VAB) (2vaB)’ (A+B-c)?- Os Segredos da Algebra para IME/ITA/OLIMPIADAS N _N-(VA+ vB VC) (418 2vaB) > eB VE (A+B-C)? —4aB N 2yFome EEE N NVA BE FEE KBE KB w____N{vA¥B VE) ° BE RB) (vey 7 Nn.(JA- vB - JC) - OBE 2h 8-e ___N(VA_¥8 WC) 9 +8-c+ 2vh8 REE RB CARB A+B C+ 2NRB ala y N (VA VB -JE).(A+B-C+ 2JAB | > TR-B-JE~ (A+B cy? -(2VAB) y N (vase) (A+B-C+2VAB) FEE (AsB-c) 488 N 4) Forma ——___ . VA ~JB~ JE N N VRB IC WEE OEE ie BE oN n-(VA- vB + JC) VRB E (jaye) (Je) Ss oN n.(VA_ WB + YE) WEE A ae ee zs NN (VR BNE) avec 28 WBE Wp Gone AB C+ 2NAB — 400 3 Racionalizaca N N.(VA- B+ VC WE) (A+8- ~ C+ 20m) > K-B Je (A+B-cy ~(2vaB) —! N N-(VA— VB +VC)-(A+ BC. Re BOE 8) VRB VC (A+B-C)* aap Exemplo Resolvido 108: Mostre que ag Bs v2 4 Bue =, Resolucdo: Podemos escrever: 1 (v2 + V8 - ¥5).(2+3-5 2473) EEE > * ays -afaane 2+ 88 -6).(-26 (v2 +8 -¥5)-(s6) p LEE) (nt 1 EB rs Exemplo Resolvido 109: Mostre que ——=—= Wr eo = =5yit- 15+18- - Resolugao: Podemos escrever: e-—__5 5-(1+ 414+ vB).(1411-5-200T1) = = S(t vtts VB). (1411-52011) _ Wie (1411-5)? 4.4.11 _5:(14 V+ v8) (7 -247) re ee | (7)? ~44 > eS 20 + 1VAi_2.11. 7482.88) aay . 5 TAT T= 15. 745 9 es Os Segredos da Algebra para IME/ITA/OLIMPIADAS 101 Exemplo Resolvido 110: Efetue ———-_—___ FOES Fe Fa" Resolucdo: Podemos escrever: 1 _ (v8-V2- Vi0).(3+ 2-104 25-2) Foe asta 1 (v8 - v2 - J10).(-5 +26) > ae Baers oF 2N2 + 52 -2- 208 + Sf -2.2415 1 - _ + - . Beara 18? 93 + 50 - 4,715. 2 _ (3v2- 42 -2/6 ~246) Vat 12 Exemplo Resolvido 111: Mostre que Resolugao: Podemos escrever: £ _ vB .(vE- 8+ 7) (443-7+244-3) OB (443-7) -4-4.3 eB BENNO BB 5 (2v2 - v6 + V14).(4v3) 0 ne sw (v6 - 14-22) -V8 Cn V2) 32 - 42 - 2v6) ao 102 3 Racionalizacag Problemas Propostos ~~ — ee " — Questao3.1 | Questo 3.2 _ Racionalize —© Racionalize 1 — RB EE Questao 3.3 (CN-1976) AVA -3N3 OT Simplifique a expressao VA-V3 a) A-9+A¥8 b) A+34J3A c) A-3JA 4) 3-A+V3 e) 9+ VK Questao 3.4 (CN-1 999-Modificada) Racionalize Be mei V5-V3 Questao 3.5 Qual 0 valor de a ? V5 - VB +2 Questao 3.6 _ ‘ v3 Racionalize ———“*____ 216 +35 - 7/2 Questao 3.7 ee Racionalize 25 . Vi2-J20 4a Z Questao 3.8 — ‘ 12 Racionalize on aE aT Questdo 3.9 _ 1 10404 Racionalize — +7, 1, 1 2 Os Segredos da Algebra para IME/ITA/OLIMPIADAS. 103 Questao 3.10 _ _ 1 1 alor de ? aval oN BR 2B Questao 3.11 jize v2 5 Racional V2- N21 8 Questao 3.12 Racionalize Questdo 3.13 / i fia ale Racionalize SY ———S— Ve + J5-2J6 Questao 3.14 5 Racionalize ~ Questo 3.15 (Moscou 1982) Simplifique a expressdo 2 . [4-395 + 25 - $25 Questo 3.16 x Qual o valor de Ya+1 7 y2- Questia 3.17 Racionalize 22=3b— vab 2va ~ 3b 3 Racionalizaca 104 Gao Questao 3.18 (CN-1983) Efetuando ,/ NS 2 a)4 ag Questao 3.19 (CN-1991) auestdo 349(CN-1991) kg. (B-1- VB Ve-1 a O valor de & PB —J/2 +1 ») 2 02 a) 242 ©) NB Questao 3.20 (CN-1994) é igual a: P 41 O numero Y2V2+3 a) V2 +1 b) V2 +2 c) ¥V2-1 d) ¥2-v2 e) i- Questao 3.21 (CN-1997) 3(v2+ V3 +V/5 +2) 1 é (Be Beep a] Peo” O valor de 8-425 b 23 +3V2 - ¥30 a + 3v2 + 4N0 12 ) —S——- a a) 24 4) E225 @) 28 -3v2-v80 12 ) 24 Questa 3.22 (CN-2012) _—_—] = et Sabendo que A = 3+ V6 a? — 3+VG 4 58 212 - Jans Jag * Wal var le Gar _ av vy MBs ) > a) "9B" oe Questao 3.23 ee Racionalize Ee B® Os Segredos da Algebra para IME/ITA/OLIMPIADAS. 105 3.3) Radicais Duplos Radicals duplos S40 radicais em soma ou difer enga de termo: transformados em algum produto notavel, S que podem ser Veremos alguns radicais duplos: 01) Radicais da forma: VA VB. Queremos transformar VA + VB_em uma soma ou diferenca de radicals. Entéo. vamos desenvolver isso: . Demonstracao 01: — seja VA~VB = m+n e VA-¥B = Ym—Jh, temos: Quando somamos: (RB NAB =n Bi = YA+B - JA-VB = 2Nm. (eat) Quando subtraimos: (e=B RAB = Sins Si (iv —W) = A+B - JA_ JB = ms Jn Jimi \A=NB - Ja_ JB = 2h. (eq2) Queremos m en, ent&o, elevando (eq1) e (eq2) ao quadrado, temos: Whe + Va BY = (adm? = A+B +2. (A +B )(4- WB) +A- VB =4m © 282. [a2 —B) <4m = 4m=2A+2- B) macht? (a? -8) At [a?-B) 4 me 2 ; NAB ABP = (ase © ANB-2. as JB)(A vB) +A- WB =4n © 74-2. (A) = an = 4n=2A~2. (A? -8) 408 3 Racionalizacag => Assim, temos: ae aa ES Geralmente, chamamos C - VA? ~B , dai essa expressdo fica: A=C, [A-c| a+vB = |= + Jace . 2 2 Demonstracao 02: Uma outra forma € enxergar como trinédmio quadrado perfeito, a saber VALB - | ai2 iB = VA+VB =vmint 2vinn man mn = VRB =n ain dain = Van = shy A= NB = \vin + vi. Demonstracao 03: Qual o valor de JA+ JB + VA—B? Seja VA+ VB -x, Ja_\B = y e E=x+y>0, podemos escrever (") Ps dy ty? =(xsyP = x? 4y? = (x+y)? —2xy = (VANB) 6 (VAB) <8? 2a B Va => A+VB+A~-JB -£2_2. A? (JB) => £2 -2A42-VA?-B = E=2(as Va? 8). B= 2(A+Va?-B)>0. ie to (*) Observacao: Esses i « itulo 5. 0" 7 Produtos not apitulo 9. os detalhes! aveis sero vistos no cap! Os Segredos da Algebra para IME/ITA/OLIMPIADAS 107 Demonstracao 04: Qual o valor de Vas +a ¥B? a a seja x= VA+VB + JA +(-VA+ Je) (AJB |=0, entao dos produtos notaveis condicionais, temos que (*): m+n+p=0 = msn? +p? — 2(mn+mp +np) = m+n? +p? =-2{m(nsp)+ np] 28 BY BY = alan) -| BY IB = x? +A+VB+A- VB =~ [ee | Jj = x? 42M =2x? -24A? -B = x?+2A - 2x2 2a? _B = x? =2A+2-VA?—B x= fof asda? -B }>0 (*) Observacao: Esses produtos notaveis sero vistos no capitulo 5, com todos os detalhes! Exemplo Resolvido 112: Determine ¥3-2y2 . Resolugao 01: Podemos escrever: ba - 32.22 = \-22 -B-8 | A-3e 8-8, c-Ja?-p = c-Ve 8 = Logo : oP fe = ¥3-V8 = 2-1. Resolucao 02: Podemos escrever: 8-28 - fave = B22 - fi- 24 +(vay > ¥3- 22 = (1-2) = 3-2/2 = [- ~ v2], como 1- V2<0 408 3 Racionalizacag Exemplo Resolvido 443: Determine 5 +/24 | Resolugao 01: Podemos escrever: Yee J24 = A=5eB=24, C= v5? —24 = OER py a - pra. Re CL. erga EE. = ~ oe Ef = JR 66 Resolugao 02: Podemos escrever: 5424 = J3+2N6+2 = 5+ V24 = (3) +2-.8 aay = ei = [B+ 8) > VERE -|5 +9), como VB+V2>0 = 15+ 24 =V3+42. Exemplo Resolvido 114: Determine 28-103 . Resolucao 01: Podemos escrever: (28-1088 = 283-10? = 28-1008 = 28 00 = A=28 e B=300, C- 282-300 = c- 784-300 => C= 484». [C=29). Logo: VaWB = JEEP Ane 6 = eee - 5% , as - |2- -§ = 28-800 - 25-8 = 28-00 -5 8. Resolucao 02: Podemos escrever: Vee -10N5 - )25-2.5.J5y3 = v28-108 = 5? 2.5 B+ (V3)? = 28-105 = fs By = 28-108 =|5 - sh], come 5-B>0 > 28-108 =5 Exemplo Resolvido 118: Qual o valor de Bi 8 + 8-8 ? Os Segredos da Algebra para IME/ITA/OLIMPIADAS. 109 Resolugao 01: Seja ¥3 + V8 =x, /3- JB ~y e E=x+y>0, entdo (*): 2 Ca dayty?=(K4y)? = xP ty? <(xeyP—any (+) = (BB) +B BY -€2-2. EE = 3+ V8 +3- 8 =E?-2. 3? -( 8)" => £7 =642.0-8 = 62 =6+2-fi = E27 =6+2 = 2-8 . E-8-22-0. (*) Observacao: Esses produtos notaveis serdo vistos no capitulo 5, com todos os detalhes! Resolucao 02: Chamando a expressdo toda de x, podemos escrever: x-¥B+ V8 +VB-VB = x-\B+J8 8-8-0. Entdo dos produtos notaveis condicionais, temos que (*): m+n+p=0 = m? +n? +p? =-2(mn+mp +np) (*) 2 => m+n +p? =-2[m(n+p) +np] = 24(- ay +[a-ey' = af x( (-x)+(-8+V8)(-W8-8)] at f?-(8) | 3246-22 -2)028 = x2= 642i = x7=8 » x= YB=2VE>0. = 2434/8 +3-V8 =- aS (*) Observagao: Esses produtos notaveis serao vistos no capitulo 5, com todos Os detalhes! 2) Radicais da forma: YA vB. Queremos transformar 3/A = VB em uma soma ou diferenga de radicais. Ent&o, Vamos desenvolver isso: Demonstracdo 01: Seja YAx VB =m+ hn e YA—VB = Quando somamos, temos: TRB Yk WB me dinem-vh > Ya ARB ~2m (et) Quando Subtraimos, temos: ewe a 0 3 Racionalizagao YB -Ya—B = m+ sh -(m- va) => Ye BR Bim YABB Queremos men, entZo, elevando (eq1) € (€q2) ao cubo, temog n, (eg (ae HR-BY my? = = 2A+3 (RBA) PAB VA B) - on? => 2A+3 (4 BIA- 8) Jam) am* > 8m? -6m [sh -06) | 24-0 * 4m? ~3m-{YR?-B)-A=0 Como Ae B s40 dados, temos que o valor pedido € racional. Continuando o desenvolvimento, note que, se subtrairmos, nao chegaremose resultado direto, entéo vamos encontrar n, em fungao de m, mulliplicande ss: duas equacées: ({ AB) AB) (meson) = (BY -w?-# Demonstracdo 02: Podemos pensar da seguinte forma: 3 (AxvBy =A £3.02 WB +3-A-(vB) + (vB) = (AxVB) = a2 13-A?.JB+3-A8 (BB ». (as sB) = 9A? 98)-A+ (3A? +B): VB Demonstracao 03: Qual o valor de YA+B +YA—vB? Seja YA+ VB =x, Ya_WB -y e E~x4y>0, podemos escrever Os Segredos da Algebra para IME/ITA/OLIMPIADAS 111 (eyo t¥ +3xv(x+y) (4) 2 -(Vanb) (UAB) 3. Yar 8 YAS 4 BI -A+NB+A vB +36. fa? (Jey > 6 -2A+3E-YA?—B ., £8 3€- Ya? —B 20-0, ) (*) Observacao: Esses produtos notaveis serdo vistos no capitulo 5, com todos os detaines!!!! (**) Observacao: Vocé aprendera a resolver equagées de terceiro grau no capitulo sobre Fatoragdo! Demonstrac4o 04: Qual o valor de Ya; JB +¥a_—JB7 seja x= YA+VB+YA-VB = x +( A+B) + (la) =0, temos dos produtos notaveis condicionais (*): m+n+p=0 => m? +n? +p? = 3mnp (*) 5A) + AAB) - 9.x (ARB) (ARB) > 8 -(a-B)- (AWB) -ax-{a? (Vy? = 0 -A-B A+ VB =3x- Ya? -B 2° ~3x. Ja? —B 2A =0. (**). ") Observacao: Esses produtos notaveis serdo vistos no capitulo 5, com todos 08 detalhes!It "y Observacéo: Vocé aprendera a resolver equagées de terceiro grau no Capitulo sobre Fatoracdo! Exemplo Resolvido 146: Determine Y0+6/3 . Resolucao 91: Podemos montar a seguinte equagao: Woo 8B ao. ee 412 3 Racionalizacao 3 (3 4m? 3m (Ya?-B|-a=0 = 4m°-3m [ 102 - 108 )-19_, => 4m? -3m-(9100-108)-10=0 = 4m? 3m (3a) 40=9 5 4m? —3m-(-2)-10=0 =» 4m? +6m—10~0, Usando 0 teorema do fator (*), note que a unica raiz Feal dessa i ha, 4m? +6m-10=0 6 1, ou seja m=1. Assim, Substituindo na Outta © me : Prey, temos: n=m? - 310-108) = n=?-3-8) => n=1-(-2) = nag, Logo: Yas WB =m+ yn = Yo+ fos =14 V3 (*) Observagao: Vocé aprendera o teorema do fator no capitulo de Fatera: Resolugao 02: Podemos escrever: fore - a +B) = (A? +38) A+ (3A? +B) vB = 104648 Por comparacao, temos: B=3 = 3A7+B=6 = 3A743-6 = 3A2-6-3 = 3A"=3 > A? =1 => A=41. (oql) eo (A? +38)A=10 = (a? +3-3)A=10 = AP49A=10. (eq2) Note que A = ~1 nao satisfaz (eq2). Portanto A =1. Logo: #10+6J3 - (14.3) => Fores =14+8. Exemplo Resolvido 117: Determine Prose . Resolugao 04: Podemos montar a seguinte equagao: We5e - frog - 4m? ~3m.(Ya?—B|-a=0 = 4m® am (7? 80 |-7=9 > 4m? ~3m-(¥43—B0)-7 =o = 4m? — 3m. (94)-7=0 = 4m? —3m-(1)-7=0 => am? +3m-7=0. Os Segredos da Algebra para IMEATA/OLIMPIADAS. 113 usando o teorema do fator (*), note que a raiz real dessa equacdo 4m? +3m-7=0 € 1, ou seja m=1. Assim, Substituindo na outra expressdo temos: n-m?-3{7?-50) = n= 34) = naa (1) > n-2 Logo: YA+ YB =m+ Jn = 7+ J50 - 14/2. (*) Observagao: Vocé aprenderé o teorema do fator no capitulo de Fatoragdo. Resolucdo 02: Podemos escrever: Ws - SA. 4B) = (” +3B)A +(3A? +B) VB =7+502 Por comparagao, temos: B=2 = 3A7+B=5 = 3A7+2=5 = 3A? =5-2 = 3a7=3 > A’ =1 > A=41. (eqt)e (A? +38) =7 = (A?+3-2)A=7 = A°+6A=7. (eq2) Note que A = —1 nao satisfaz (eq2). Portanto A =1. Logo8 Wes a8) = F752 -14+V2. Exemplo Resolvido 118: Determine {45 - 29V2 . Resolucao 01: Podemos montar a seguinte equagao: Ves258 aso 20° - sree] 4m? — 3m (¥a?-B)-a-o => 4m? - am-{ Yas? 1682 ) 45-0 % Aa. ( QEODEASED) 45-0 > 4m? ~3m-(¥543)-45=0 © 4m? —~3m.7~ 45-9 = 4m? -21m-45=0. Usando teorema do fator (*), note que 2 raiz real cesse. eauaeie 4m _ 2m-45=0 €3, ousejam=3. 3 Racionalizaca 114 Gao Assim, substituindo na outra expressdo temos: n=m? | 5? 1682) => n=3? -%843 > n-9-7 _, nop Logo: an YB -m-yn => ¥Ya5— Jiea2 =3 2 (*) Observacgao: Vocé aprendera o teorema do fator no Capitulo de Fat ray Resolugao 02: Podemos escrever: —e 3 as one - (av) = (A? .9B)A (30? +B) VB ~ 45-205, Por comparacao, temos: B=2 > 3A7+B=29 > 3A7+2=29 = 3a? =29-2 = 342_»y = A?=9 > A=33. (eqi)e (A? +38) A = 45 = ( +3-6)A=45 = A346A=45. (eq2) Nole que A = -3 ndo satisfaz (eq2). Portanto A =3. Logo: 445 20 - 43-2)" = Yas—29,2 -3-V2. Exemplo Resolvido 119: Qual o valor de 410+ /708 + 310-108 ? Resolugdo 01: Seja 104/108 =x, Yio—Jioe -y e E=x+y>0, ten dos produtos notaveis (*): 3 Dt yP =P ry? + 3xy(K4y) = & = (Fo Fda)’ «(ode «se. fro srow. roe _ 3 Se ~10+ 106 +10 08 + 96. tho? -( fio)’ a 3 E° =20+3€.3700—108 = E2 =20+36.97B = E? - 20736 (2) = E=20 6 - © 406-200, (**) Note que, ; 7 ail Lopes Belo teorema do fator, 2 ¢ raiz dessa equagao de teroeir© gat (1) Observagao: voll os detatnent * ESSes Produtos notaveis serdo vistos no capitulo 5: Os Segredos da Algebra para IME/ITA/OLIMPIADAS. 115 (rt) observacdo: Voc® aprender a resolver equagdes de terceiro grau no capitulo sobre Fatoracao! Resolucdo 02: Seja > Ye FE Poe «(Bo Fes) | io aa) -, entdo dos produtos notaveis condicionais, temos que (*): minep=0 > mens +pe=3mnp > vf4[ Mio) +( FO 0) 9x. (Ov FOB | -Sho- F08) -5 (10 + 0B) - (10-765) 3x. fo? (8) = 8-10 ¥108 - 10 + 708 = 3x. 100-108 > x9 -20=3x- YB => x? -20=3x-(-2) x9 +6x-20=0.(*") Note que, pelo teorema do fator, 2 é raiz dessa equagao de terceiro grau. Logo: x = 2. (") Observagao: Esses produtos notaveis sero vistos no capitulo 5, com todos os detalhes! (") Observagao: Vocé aprendera a resolver equagdes de terceiro grau no capitulo sobre Fatoragao! 03) Radicais da forma {A+B . Para tadicais com indice 4, basta fazer radical duplo de indice 2, duas vezes. Vejamos alguns exemplos: Ls m xemplo Resolvido 120: Mostre que 4/1614 72V5 = 2+ VB - Re: Solugdo: Podemos escrever: ~—_ Vi61. = 2 725 = 48142.9.4/5+80 = 4161+72V5 = {(9+4V5) Com - 9445 50, he14 725 - (9+4v5 118 3 Racionalizagag Jo. 4J5 - f412-2N2+5 = era - [2 F como 2+V5>0, 4161+ 72V5 = 245. Exemplo Resolvido 121: Mostre que Yi9—1328-3 5 Resolugao: Podemos escrever: {froa—132N2 = $f121-2.11-6N2 +72 > roa 1328 = (11-642) > ise - fre como 11-6y2 >0, 4193-1322 = 16/2 Vi-8B e282 = 68 - s—e) = V11-62 = [8 - | como 3-¥2 >0 = Yiss—1322 =3-V2. Até agora, vimos radicais para cuja resolugao usamos as formas (a+b? (a +b)> e(a +b)4 . Vamos ver agora aqueles em que usamos as formas (a+b+cfe (a+b+c)*. Vamos la: Como (a+b+c)* =a? +b? +0? + 2ab +2ac + 2be , podemos fazer a=\m, b=Jh, c= Jp, dai fica: (Vin + + py =m+n+p+2¥mn +2 Jmp + 2Jnp| * ((vm+ Mis JB) ~m+nip+Jamn + Jamp + amp Ent&o temos (04) Radicais da forma; VANE IE+ We VA+VB4JC44D = m+n+p+2vmn+2/mp +2\np a > VA+VB+JC4.D = m+n+p+Jamn + Jamp + /4P Os Segredos da Algebra para IME/ITA/OLIMPIADAS 117 2 BEB - [ii oP onde: A=m+n+p, B=4mn, C=4mp, D-4np. Exemplo Resolvido 122: Efetue y10+ 2/6 + 2/10 +2415 Resolucao: Podemos escrever: (id: 206 + 2010 + 215 = f2+3+5+ 22 -13-+ 212. V5 +235 Note que m= 2, N=3, p=5. Logo: 2 fio 206+ 210 + 2N15 = y[ V2 + V8 + VB) Yi0+ 26 + 2y10 + 2015 = V2 +3 +5. Exemplo Resolvido 123: Efetue 37 + 12v6 + 614 +4V21 Resolucao: Podemos escrever: b= sar tye +6 fia Aa = €-,)7+29)-3.42} [5)+2-3-02)- 7 +22) 7 > E= 3742.42.32 v3.22] +2. y2-3"| fea bbzlv SE- 18+124+7+2. ¥78| +2- Questao 3,33 < Be Qual o valor de 134/48 > Os Segredos da Algebra para IME/ITAYOLIMPIADAS 119 questao 3.34 ‘Qual o valor de 291205 — 29125 7 Questao 3.35 ge a igualdade ‘eo = 2y2x = 11+ 3V8 é satisfeita, determine o valor de x. Questo ¢ 3. 36 __ Determine o valor de Mem 3 E 5 Questao 3 3.37 Determine 0 valor de ¥39- 1213 + as 23 Questao : 3.38 (CN-1984) Aexpressao > ae 2 Yai gave —3- 2% 2 é igual a: a)1 b)2 o)3 d)4 e)5 Questdo 3.39 Qualo valor de 43 —12V7 - 43+ 12N7 ? Questao 3.40 Qual o valor de 632007 - ¥53+ 207 ? Questo 3.41 Qualo valor de 57 —40V2 - {57 + 402 ? Questéo 3.42 Qualo valor de J172—96V3 ? uestio 0 3, 3.43 Qual g © valor de 324107 2 a2 1047 + (32-107? 120 3 Racionalizacao Questao 3.44 ( 3.44 (Princeton-2006) — = Simplifique ue V7 4h + +7 43 . — Questao 3.4 3.45 (AMC- 2011) Qual dos valores abaixo € igual a Jo-6V2 + 1a 0-642 + fo-6 - ie a) 3V2 b) 246 = d) 343 ae Questdo 3.46 (AHSME-1970) O numero /3+2V2 -v¥3- 2V2 é igual a: a)2 b) 2V3 c) 4v2 d) 6 e) 22 Questdo 3.47 (IMO-Longlist-1988 - Modificada) Calcule o valor de x em X = (tev) vit 62 =(1 ANITA Vir-6W8 -(11- 662) sitet (OB 2 IB) Questao 3.48 (AHSME-1976) _ V5 12+ VV5=2 _ 5-9), entio N é igual a: Se N= v5 +4 a1 b) 242-1 of d) e e) NDA Questao 3.49 _-— Questo349 Qual o valor de Y2 + V5 ? _— Questao 3.50 (CN-1982) ~ O valor de {10+ 6V3 é: a)1sV7 by) 148 co) t4V5 a) 148 Questio 3.51 (CN-2011) eee O numero real {26—15J3 é igual a: ei” _ Dia 3 qd) fis-38 e)2 Os Segredos da Algebra para IME/TAOLIMPIADAS 124 questao 3.52 (Stanford-2008) —_ 77 +4 simplifique pom 7 Questo 3: 3. 53 (IME-0; 02/03) ¥f a Demostre que 20+ 14Y2 + 20142 6 um numero inteio Miltiplo de 4. Questo < 3.54 Mostre que » ee 185 15¥3 + 26155 eum: +153 6 um numero inteiro. Questao : 3.55 (Turquia-2007 -Modificada) Determine 0 valor de BN ee a Questao 3.56 (AHSME-1980) _— Asoma J5+2V13 + ¥5— 213, é igual a: 3 Yes 14913 a)5 | I> a 2 e)1 Questdo 3.57 (Turquia-2009-Modificada) Determinando o valor de x = 911+ J337 +411 V337 , quanto vale x 3 18x? Questao 3.58 Qual o valor de Y5J2+7 —Y5V2-77 Questo 3.59 (IMO-Longlist-1973) Ontmero Yee 5 mero YV/5 +2 + Y/5 — 2 , 6 racional ou irracional? Q —sta0 3.60 (Suécia-2001) a Most s 4 We que ( Y24.5)5 +(J52—5)8 ¢ irracional. 122 3 Racionalizacag Questao 3.61 (Malasi : 1 =~ Mostre que existem inteiros m e n, tais que _ 50 4 7S Questao 3.62 (Ira- -1989) 1 vr? 413 ‘ . Mostre que [m4 in? +4 nh vet € um numero inteir q © Positivy x. m(m* +3 . - (me? para algum m inteiro positivo. n= Questao 3.63 > 3.63 (IME-90 90/91) pe Mostre que o numero pe pe 1 +— {o-p-= as = €racional, Questao 3. 64 (CN-2004-Modificada) Simplifique e {a9 +206 . Questdo 3.65 Qual o valor de ¥8+/40 + /20 + V8 ? Questao 3.66 (Kosovo-2013) _ Prove que que Ji0+ Vat a0 SO V2 +8 +5. Questo 3.67 (AIME-2006) See ME 2a — , Onumero J104/6 + 468/10 + 14415 +2006 pode ser escrito com? “9 abs. aV2 + bv8 + cVB,, onde a, b ec sao inteiros positivos. Determine 2 Questao 3.68, ene Questao 3.¢ ee Qual o valor de 428 +4 Jae 2 Questo 3.69 _ TT Qual o valor de {28 — 4. Jag > Os Segredos da Algebra para ME/TA/OLIMPIADAS 23 uestdo 3.70 qual o valor de {17+ 12/2 > Questdo 3.71 (AIME-1990) 3 3 Determine (52+ 643)? - (52-643)? . questo 3.72 (CN-2003) ge a= y4—-Vi0 1 2¥5 © b= y44 V0 215, entioa +b igual a a) 10 b)4 c) 2v2 d) V5 +4 e) 342 424 3 Racionalizacag 3.4) Topicos Avangados ‘Nesse topico -avancado veremos como se comporta um } aa ical dag indice 3, em cuja resolu¢do usaremos (a+b+ oc)’. Vamos |a: 0 ep Desenvolvendo (a+b+ cy , temos: (arb+c)=a"4 p?+0? + Sab? + Sac? + Sab +3476 +3676 4 3be? 4 gate podemos fazer a= Ym, b= Yn, c= ¥p , dai fica: , (QW + 3) 2x4 x8 4x? + 5x—11=32-8-16410-11 = 2x4 x8 4x2 45x 41-7, Os Segredos da Algebra para IME/ITA/OLIMPIADAS 127 43) Operagdes com as Expressoes Algébricas @ Adicao de Polinémios — Aadigao de polinémios é efetuada pela soma dos termos semelhantes (termos com @ mesma parte literal). Conserva-se a parte literal e somam-se os coeficientes. exemplo Resolvido 125: Efetue xy? + Sxy + 3x3y2 Resolucao: Note que temos duas partes literais iguais. Conservamos a parte literal e somamos OS coeficientes, 0 resto se repete py? + Sxy + 3x89? = 4x°y? + Sxy- Exemplo Resolvido 126: Efetue axty + 3xy? +22? + ox4y + 3xy3 4.92241 Resolucao: Note que temos partes literais iguais. Conservamos a parte literal e somamos os coeficientes, o resto se repete. axty + Sxy? + 227 + Sx4y + 3xy? +927 +1 = 8x4y + 6xy3 41122 44. b) Subtracao de Polindmios A subtragéo de polindmios € efetuada pela soma dos termos semelhantes (termos com a mesma parte literal). Conserva-se a parte literal e subtraem-se os coeficientes. Exemplo Resolvido 127: Efetue -3xy? + 24xy? ~32x?y—4x7y Resolugdo: Note que temos duas partes literais iguais. Conservamos a parte literal e somamos os coeficientes, 0 resto se repete. Bay? + 2axy? —32x?y — 4x2y = 21 xy? -36x7y. Exemplo Resolvido 128: Efetue xyz — 2xyz + 2ab - 5ab —mn+ 7mn. qesoludo: Note que temos partes literais iguais. Conservamos a parte literal Mos os coeficientes, o resto se repete. %E°2ye + 2ab Sab —mn+7mn = 6mn ~ 3ab - xyz- hee a ineina é inamio deve multiplicar 0 temmean, dois Polindmios, cada termo de um polindmio eve ee 0 outro polinémio, propriedade que chamamos de “distributiv: — 128 4 Expressdes Algébr, cas Lembrando que multiplicamos os coeficientes e as partes lit S liter No final, se obtivermos termos semelhantes, somamos oy ‘ais de su 4 btraimoe a operagao aparecer. aims Coe in Vejamos oS exemplos: Exemplo Resolvido 129: Efetue (-Sxy)(+4x°y?). Resolucao: Note que temos duas partes literais. Conservamos a p; somamos os coeficientes, 0 resto se repete are eras (-Bay)(+4x8y2) = -1ax4y"? => (30) +4x5y?) = 12x83 Exemplo Resolvido 130: Efetue (- 7x9) (x4y7z — 2xyz°) Resolugdo: Conservamos a parte literal e somamos os coeficientes, o restos. repete. (-7 \[xtv’z — 2xyz2> (-78)(x*y’z ~ 2xyz*) =~ 7x7 y2z.+14x4y2°. 7x3*4y?27 +7 e2x3t yz Exemplo Resolvido 131: Efetue (ab? +2abe? )(a?e -b*c) mos 4 parte iter? Resolucdo: Note que temos duas partes literais. Conserval somamos os coeficientes, 0 resto se repete: (ab? +2abc?)(a’c -b4c) = ab? (a’e -b‘c) +2abo? (a (ab? +2abc2 Jee -b*c) ~a'*2p2c _ab24e+ 2a'*2po?*" —2al > ; (ab? + 2abo?)(a?e ~b*c) = a%b®c—ab®e + 2a3bo? — 2ab°e°. 2c -btc) pitt , 2,2 Exemplo Resolvido 132: Efetue (xy - 2yz+ 2x2) (x7? +x2z?-¥? ) pes Conservamos a parte literal e somamos OS coeficiente> ° = 3 (19 -1)(8 V8 4 2y? ay 6) ty’ BYE —x2y? + 6a Ro _y2y2+ny-F (xy—1)(x3y3 5 2, (y-1)(ey? + ¥° ~xy+6) xty4 —292y2 xy 8. Os Segredos da Algebra para IME/ITA/OLIMPIADAS 129 d) Diviséo de Polinémios Na diviséo de monémios, efetuamos a divisé 0 dos coeficientes (qu: possivel) € efetuamos a diviséo da parte literal (evando observa¢ao: Aqui, irei apenas citar a divisdo de Mondmios, para nao fugir dos objetivos deste livro (+24x°y?) “C30 Resolugao: Note que temos duas partes lterais. Conservamos a parte literal e somamos 0S coeficientes, o resto se repete. Sy? 5,2) (28°F) 224 (sya), (2¥) (-3xy) -3 (3x) Exemplo Resolvido 133: Efetue =-Bx4y. 5 Exemplo Resolvido 134: Etetue 3%] (Bxty?2], Resolugao: Conservamos a parte literal e somamos os coeficientes, 0 resto se repete. Coeficiente: 8 A Parte Literal = t= xty?zy*z (5,8 2x4 3y*z __ Problemas Propostos oe Questo > 4. 4 | (CN~ -1952) Hetue @ multiplicagao (x Questo 4.2 (CN-1952) oe Smpliique a expresso 16x" — y psx? “« sy): voy MN x 5 Produtos Notaveig Capitulo 05 - Produtos Notaveis Introdugdo Frias @xpiseahee ‘No caloulo algébrico, existem varias expresses algébricas (ou pain uso € bastante frequente em fatoragGes e simplificagdes, essas, 8) - xpress “te chamadas de produtos notaveis. Neste capitulo, vamos, estudar ferramentas importantissimas e muito eficazes nas Simpliicagges = ; & express6es algébricas. 5.1) Quadrado da Soma de Dois Termos Scuadado da soma de dos lemos Sa O quadrado da soma de dois termos € igual ao quadrado do Primeiro tem mais 0 dobro do produtos dos dois termos, mais o quadrado do S€gundo tem, (a +b)? = a2+ 2ab + b2 Demonstracao: (a +b) = (a + b)(@ +b) & (a + bP a24 abs abot? . (a+b)? = a2 + 2ab + b2h Exemplo Resolvido 135: Efetue (2x +3)?. Resolugdo: Podemos escrever: 2 (2x +3)? = (2X+3)-(2x+3) <> (2x+3)? = (2x)? 42x-343-2x+3 © (2x43? = 4x2 s6xsex4g - (2x43)? = 4x? 12x49. Exemplo Resolvido 136: Efetue (x42). Resolucdo: Podemos escrever: (+27 = (x42). (42) (x+2P = ° x2 4x 242-x +22 2 (x42) =x? 42x 42%44 (x+2)? =x? 44x44, Os Segredos da Algebra para IME/ITA/OLIMPIADAS 131 Exemplo Resolvido 137: Efetue (4x + 1° Resolugao: Podemos escrever: (ax+t} =(4x+1) (4x41) <> (4x41? = (4x)? 4 ax 444d B 2 (axed t) = 16x? + 4x + 4x41 (4x44)? i +Bx +1. 6.2) Quadrado da Diferenga entre Dois Termos ‘O quadrado da soma de dois termos € igual ao quadrado do primeiro termo, menos 0 dobro do produtos dos dois termos, mais 0 quadrado do segundo termo. 2ab + (a Demonstracao: (a - b= (a -b)-(a-b) 2 (a — by 2ab + D’ (a - b= a?- ab - ab + b? Exemplo Resolvido 138: Efetue (3x -2)” Resolucao: Podemos escrever: (3x-2)? = (3x-2)-(3x-2) = (3x-2) © (8x-2)° 9x2 -6x—6x=4 ~. (3x2)? = 9x7 -12K+4. (3x)? -3x-2-2-3x +2? Exemplo Resolvide 139: Efetue (22° -b). Resolugio: Podemos escrever: E-[3e* 42)? = E=(30° -b?)-(3a? -b?) = 352 +b* =(36°)° 298 2p? 30 +(b?) > € = 9° ~ 32°? —3aPb" +P ° (20° 2)? = 0a° — 62°? +b’. Bem, 2 Plo Resolvido 140: Efetue (a -2") - Produtos Nota 432 5 veis Resolugao: Podemos escrever: e-(an-2y = &=(8"-2") (2 e-(a)-2” 2 2a +(2°) = E=a™™_2.a". on, om m2) 5 ~ (am ry = a2 _pnitgm , 22m. 5.3) Identidade de Legendre para a Soma “soma dos quadrados da soma e da diferenga entre dois termos 8 igug > dobro da soma dos quadrados de cada termo. a (a + by + (a ~ by = 2[a? + v?] Demonstracao: (a +b)? + (a - b= a? + 2ab + b?+ a? 2ab + b? © (arb) +(a-b)? = 2a? +20? . (a+b)? +(a-b)? = 2(a? +b?) 2 2 Exemplo Resolvido 141: Mostre que (7a +b?) + (va ~b?) = 2(49a +0!) Resolugao: Podemos escrever: E =(ras02) +(a-v?)? SE = 27a) +) B= 2(49a? +b). Exemplo Resolvido 142: Mostre que (1+ 2) +(1 - y = 21+ *) Resolugao: Podemos escrever: E-(tHef +(e) > e=217+(8)'] = E=2(1+%°). Exemplo Resolvido 143: Efetue (9 p™ +4) +(9p” -4). Resolucao: Podemos escrever: E=(9p 4) (apa? = E=2|(0pnf +4? ~ E=2(81p™" +16). Os Segredos da Algebra para IME/ITA/OLIMPIADAS 133 tidade de Legendre para a Diferenca sa) iden drados di Giorenga ene 0S quadrados da soma e da diferenga ae wy quatto vezes 0 produto desses dois termos, $8 entre dois termos 6 igua fa by (ab) — aan] pemonstracdo: am 2 2 (a + bY (a= by = a? + Zab + bp? (a? 2ab + b?) es e@ (a+ b)- (a ~ bY = a? + 2ab +b? a2 2ab — be (a+ bP ~ (a — bP = aan), , 2 Exemplo Resolvido 144: Mostre que (x* +1) -(x* a y 4x4, Resolucdao: Podemos escrever: oan (x4 wap (x4 a)? = 4x4 ety (x* a1) 4.x =4abe. Exemplo Resolvido 145: Mostre que (ab+c)* ~(ab-c Resolucao: Podemos escrever: (ab=o)* -(ab-c)* =4ab-c -. (ab+ co) ~(ab-c)* =4abe. 2 2 Exemplo Resolvido 146: Calcule (a2o® +e™a" J -(a?b* -o"d") . Resolugao: Podemos escrever: 23 2 (ab 2") (ao? -ona" =4(a°b?) -(c"4") le Lagrange para a Soma: izar a soma de dois quadr: (ax + by)? + (ay - bx)? 55, @ 6 resultado sera: ae (xe + ¥*) ta _ —_ 5 Produtos Notéveig Demonstracdo: E=(ax+by) +(aY ~bx) ; JF 4 2-ax-by + (by)? + (ay) ~2-ay-bx + (bx)? = E=(™ 2 2 (2 2 2 = oy B= ab? + bey? + ay + bx? > E a(x +¥)sb°(y? 2) 2) 2 - fax + byy= (ey - bx) = [2?+ be)xt= y*) 7 2 Exemplo Resolvido 447: Mostre que (2x +3y) +(2y-3xy hy Resolucdo: Podemos escrever: 2, 32 |[y2 4 y2 E=(2x+3y) +(2¥-3x)) => e-|2 +3 (Pav?) Eat? J fem. an\?. fam _ane\? Exemplo Resolvido 148: Determine (5 x49! ) +(5 -3' x) . Resolucgdo: Podemos escrever: (s"x+3" y +(s -3°x) -|("y (") +?) ‘ (smx-3r) +(5" -3"x), = (52 +3")(x? +1). Exemplo Resolvido 149: Calcule {2-p? + = -«) « (3 7 c Resolugao: Podemos escrever: 5.6) Identidade de Lagrange para a Podemos generalizar a diferenga Os Segredos da Algebra para IME/TAVOLIMPIADAS 135 pemonstracdo: e-(ax+by) ~( (ax)? + 2-ax-by +(by)? —[(ay)=2-ay be+(bx}'] ay +bx)° a E= = Eea'x? +2-ax by +b?y? —a?y?~ 2-ay bx —b?x2 oy B= ax + Dy? ay? — b?x? => E ~a?(x? ~y?) b? (x? -y2) JO? - ¥?) [ax ~ by) — (ay + bx) = (3° exemplo Resolvido 150: Mostee que (4b + 3c)? -(3¢ - 4c)? =7(0 -2). Resolugao: Podemos escrever: E=(40+3c)" -(3¢ 4c >E 2 -% |e? -c?).. E=7(b? -c?) 16 9 ° Exemplo Resolvido 151: Determine (ab*Ve +3yvK) —(ab?vK+ aye). Resolugdo: Podemos escrever: E=|ab°VE + ayy) -(ab* VK +3ve) lw?) ont] (ef C4] » L Exemplo Resolvido 152: Determine 2 (a?009,2010 | 201 gore? (arora? 4920102011) Ri 5 *Solucdo: Podemos escrever: 2 44 & =(92009,2010 +c2011g2012)? (ar00earot? p20: ) Fe -l[em y -(c2" y rey (@)| - E=(q4018 ~ 08022) (y4020 _ 4) . 136 5 Produtos Notaveis ————~Problemas Propostos oe Questao 5.1 (N 54 (Noruega-1999) ee 777? - 68° . Compute “777 + 66 Questao 5.2 (Noruega-1999) y=63, determine x24 y2_— ~~ Se ny -6 Xt xy +X+ —— Questao 5.3 (Noruega-1998) Sejam a>b numeros reais, tais que a? +b? =31 ab= 3. Entao, quaniovas a-b? Questao 5.4 (Harvard-MIT-2012) Sejam ae b numeros complexos tais que 2a+3b=10 e 4a? + 9b? = 20, determine o valor de ab. Questdo 5.5 (AHSME-1958) 1 se xy=be b+ i, determine o se ae fs So as raizes da equagio 3x2 4.x ae tel valor de a B Questao 5.11 (AHSME-1951) Seres sao as raizes da equagdo ax? +bx+¢=0, determine 2 2 2 b* -4ac b* - 4ac - ———— ¢)- a) b° — 4ac 3 ) a e) b? — 2ac ae Questo 5.12 (Hungria) , 2 2 ge Sejam a, b, ¢ @ d niimeros reais tais que a°+b?=1 e c?+d°=1. Se + bd = 8, determine o valor positivo de ad—be . Questia 5.43 13 (Harvard/MIT-2008) sas, “eine g Gue a, b,c, d so numeros reais satisfazendo a> Fig “1b? +e? =1¢ ac+bd = 2. Determine ab -cd. Que, ~ “Ho 8.14 (Eotvis- 1933) sc +bd=0, Snag nc ad? =1. Se ac+bd=0, tem ed numeros reais tais que a + N® 0 Valor positive de ab-cd. — 138 5 Produtos Notaéveig Questdo 5.15 (india-1998-Modificada) oo a?) a an? Dae Mostre que (2 + 3p? |(c? + 3d’) = (ac + 3bd) +3(ad—be)2 Questao 5.16 (Hong Kong-2002) Ss Sejam x4, X2, ¥1 & Y2 numeros reais, satisfazendo as equagdes ¢ +88 2 XV1—Xiy2 =5 © x4y4 + 8X2V2 = Vi05 . Qual o valor de yj +5y3 Questdo 5.17 (Eslovénia-2010/ Kosovo-201 3) : fe, tok a+b a-b Sejam a e b numeros reais, tais que al # |b] e ——+ = | aue =P] © Soar 3b? a? —b? Determine 2 +5", @ —b" a—b? a? +b? Questao 5.18 (Junior Balkan-1997) 22 2, 2 8 Dados x e y reais com — +45 =k, determine x* Keay Xe sy’ ¥- termos de k. Os Segredos da Algebra para IME/ITA/OLIMPIADAS 139 oma de dois quadrados pode ser escrita de duas As a de quadrado da soma, e€ a segunda é ¢ form diferenca. maneiras: a primeira é em m forma de quadrado da 5,7) Soma de Dois Quadrados em Forma de S primeira escrita é em forma de quadraco da som 2 + be = (a +b)? Dap] Demonstracao: a+ 2ab+b?= (a+b? ~ fo? + b?— ay by oan Exemplo Resolvido 153: Reescreva 49 + x? como quadrado da soma. Resolugao: Podemos escrever: 494x2 = Tax? => 494x2 =(7 4x)? -2.7-x 494x2 = (74x)? - 14x. Exemplo Resolvido 154: Reescreva 16x? +121 como quadrado da soma. Resolugao: Podemos escrever: 6x? +121= (4x)? 4412 => 16x? +121= (4x +11)? -2-4x-11 16x? 121= (4x +41)? 88x. a ) Soma de Dois Quadrados em Forma de Diferenga: Segunda escrita é em forma dé quadrado da diferenga. )? + 2ab la? + b? 2 2 Sem 8 i Plo Resolvido 155: Reescreva 4a‘b” + & como quadrado da dife renga. 140 5 Produtos Notaveigs Resolugado: Podemos escrever: (4)? c= tatt? + = E-(2%) +5 2 / ff 4 cf a2 0 => E =| 2a%b- =| +2-20°b- ” E-|a%-$ babes Exemplo Resolvido 156: Reescreva 9x? + 25y49 como cuacia 00 ¢, diferenga. 0 Resolucdo: Podemos escrever: 2 2 E-9x? s25y"9 > E = (3x) +(5y"8) => 2 2 E = (3x? —5y24) +2-3x? 5y*4 B= (ae - sy?4)" «30x? -y"4 5.9) Produto da Soma pela Diferenga: O produto da soma de dois termos pela diferenga desses mesmos dois termes € igual ao quadrado do primeiro termo menos 0 quadrado do segundo terme. (a +b) (ab) = a?- b? Demonstracao: (a+b)-(a—b)=a?-ab+ab-b? ~. |(a + b)-(a - b) Generalizagées do Produto da Soma pela Diferenga: ___-- a0 Podemos generalizar o produto da soma pela diferenga entre dois termos, veremos a seguir. a) Bindmio Soma e Diferenga com Pt i oténcia oe? © produto de um binémio soma pelo bindmio diferenga ¢ igual 2° qua? Primeiro mondmio menos o quadrado do segundo monémio. (a + b°)-(am — be) = a? - pb Demonstracao: (a t br) -(am _ b") = a2™_ gm pn, am pn _ bp we) ae 0s Segredos da Algebra para IME/ITA/OLIMPIADAS 141 Generalizagao para uma Poténcia n. roduto da diferenga pela soma das poténcias de dois termos em PG é igual ° e ga entre quadrados com a poténcia consecutiva. a diferen (eo) (a+b) (a? +b?)-(a4 +b*) oo (a +b)=a?" ~ er Demonstracao: g-(a- »)-(a + b)-(a? + v?)-(a4 + b’). (a2? + vr) i” e-(e - b?).(a?+ b).(a4 + bf)... (a? + vn) a ee(a! -p').(a4 +b4)....(a" +b") 2 £=(8 -p?)....(a +b)=.. . (a-vy(er)(@ +b?).(a# eb)... (a? +p) =a?" ~ pe Exemplo Resolvido 157: Efetue (2a + 3b) -(2a— 3b)- Resolugao: Podemos escrever: (22+ 35).(2a-30) = (2a)? -(3)? (2a 3b)- (2a 3b) = 4a - 97. Exemplo Resolvido 158: Efetue (m + p%) (mm - p%). Resolugdo: Podemos escrever: (n?-9p8)(r? —p8) = (P)? —(p2)* = (rm? +p)? ws) =<" Exemplo Resolvido 459: Efetue (x—2)-(x+2):(7 +4) (0 +2). Resolucao; Podemos escrever: ES(x-2).(x42).(x? +4)-(x4 +16)-..- (2 +2") $ Ex(x 4) (12 + a).(x4 +16). (er +2") © (at 16). (x 410). ("+2") 2). (x42) (2 +4) (ite) (oanezen) 2 a 5 Produtos Notaveis ora os produtos notaveis que tem relagao com ag amados de identidades de Stevin. SA0 pouco usa, io identidades importantes numa prova qu Veremos agi grau. S40 ch s las hy simplicidade, Mas sé S devigg , ° © exija range 5.10) Identidades de Stevin para Dois Termos: “Ul a) Produto entre dois binémios soma, com termo comum: ——~ [xv ayix sb) + (a+ Bx a 2 Demonstraca (xr a)(ceb)= 2 bx Fax Fab b) Produto entre um binémio soma e um bindémio diferenga, com tems comum: (x + a)-(x — b) = x? 4 (a — b)x — ab Demonstracai (x+a)(x—b)=x? —bx +ax—ab (x +a)(x—b) = x? + (a—b)x-ab c) Produto entre dois binémios diferenca, com termo comum: (x = a)-(x - b) = P= (a + b)x + ab Demonstraca (x-a)(x—b) =x? —bx-ax+ab-. |(x a)(x—b) = x2 —(a+b)x=2 Exemplo Resolvido 160: Efetue (x +2)-(x+3). Resolucgao: Podemos escrever: (24+ 2)-(x+3)=x2 4 (243)K 42-3 o (42) (x# 3) HX? +HxPF Exemplo Resolvido 161: Efetue (x +ab)-(x bc) - Resolugao: Podemos escrever: (x+ ab) (x-be) =x? + (ab —be)x —ab- be ~ (+ ab). (x—be) = x? +(a—c)bx —ab2c, Os Segredos da Algebra para IME/TA/OLIMPIADAS 144 emplo Resolvido 162: Efetue (x abc)-(x bod) ResoluGao: Pademos escrever: (x abe): (bod) x? (abe + bed) x + abe bed (x-abe) (x—bed) = x? (a+ d)bex » ab?e7a consequéncia, pate x = 1, temos: e substituirmos x = 1 nas identidades de Stevin, teremos identidades bern interessantes, vejamos: (ta) (1+b)=P +(atb)-trab « [(1+a)(1+b)-asbvabet a) p) (tea) (1-b)= 1? +(a—b)-t-ab [vay (1 by ab ab] o) (t-a)-(1-b)= 12 -(a+b)-t+ab ~. (1-4) (1b) aba bef] Generalizacées das Identidades de Stevin: Podemos generalizar as identidades de Slevin, como segue. — a) Produto entre dois bindmios soma, com termo comum: (x" + a) (x + b) x4 (a + bx" + ab Demonstracao: [e+ a)-(x™ + b) = x27) bx™4 ax™ + ab [ers a) (xb) = x2 (a 4 b)x™+ ab}. 4) Produto entre um binémio soma e um bindmio diferenca, com termo comum: =—_ i 2} ie 5 Produtos Notaveis 144 roduto entre dois bindmios diferenga, com termo Comum: c) Pi 3 5 (x= a] [x"- >] =x (at b)x™ + ab] Demonstracao: m + ab (xr a}-(x"- b) = x77 bx® — ax’ Exemplo Resolvido 163: Efetue (2x” +3): (2x” +5). Resolugdo: Podemos escrever: 2. (2x1 +3)-(2x7 +5) =(2x7) +(8+5)-2x7 43-5 (2x7 +3)-(2x7 +8) =4x"# + 16x7 +15. Exemplo Resolvido 164: Efetue (2016 + 2b°).(a?01° -3e4). Resolucao: Podemos escrever: (a?0"6 « 203).(22018 3c) = (ar6y? +{20° — 3c }a20'8 - 20° act 2. (azote +208) (ae0"8 ~3c4) = 4032 +(20° 304 )arore _ epee’. Exemplo Resolvido 165: Efetue (p™ -a® ) {o"” -b! ] . Resolug&o: Podemos escrever: a A (nr? (0 at}. (p" bY) =(o"") +(a* +b¥)-p™ -a%b? “2 (o-09)-p? fora) n m+ aXb’. Os Segredos da Algebra para IME/TA/OLIMPiADAS 145 De um modo geral, podemos escrever as identidades de Stevin das seguint uintes maneiras oe 4) Produto entre dois bindmios soma (genéricos) com termo comi 1) ~ om? - lum: (ax +b) (ox 4d) = ace? (ads boys pd Demonstra¢ao: ° fax + b)-(Cx + d) = acx + adx + box + bd [ax + b)-(ox + d) = acx? + (ad + be)x + bd b) Produto entre dois binémios diferenga (genéricos), com termo comum: (ax_= b)-(cx = d) = acx? ~ (ad + bc)x + bd Demonstraca (ax - b)-(cx - d) = acx?- adx - box + bd » (ax - b)-(cx — d) = acx®— (ad + be)x + bdf. ¢) Produto entre um binémio soma e um binémio diferenga (genéricos), com termo comum: (ax + b)-(cx — d) = acx?+ (be — ad)x - bd Qemonstracao: (& + b)-(cx ~ d) = acx? - adx + box - bd ee , [+ d)-(ex ~d) = acx?+ (be — ad)x - bd}. 4 / . ” Produto entre um binémio diferenca e um bindmio soma (genéricos), ©0m termo comum: (ax —b)-(ex + d) = acx?+ (ad - be)x - bd “tion traca etetade ~b i Hox + a) = acx? + adx — box - bd (a < )x = bd]. SV (ex +d) = aox? + (ad — be vO 146 5 Produtos Notaveis Exemplo Resolvido 166: Efetue (2x +5).(3x+ 4). Resolugao: Podemos escrever: 8 15) BhcieD Wek (2x+5) (3x44) =2-3-x? + é (2x +5)-(3x+4) = 6x? + 23x +20. Exemplo Resolvido 167: Efetue (x —3).(7x-2). Resolugao: Podemos escrever: 21 (x-3)-(7x-2)=1-7-x nfs 243-7 |.x4+3-2 Lm _ (x=3)-(7x-2) = 7x? 23x +6. Exemplo Resolvido 168: Efetue (Sa +2).(9a—3). Resolugdo: Podemos escrever: 18 15) (5a +2).(9a-3)=5-9-a* seo 8:3 ha 2:3 . (5a+2)-(9a—3) = 45a? +3a-6. Exemplo Resolvido 169: Efetue (ax — 4).(3x +b) - Resolucdo: Podemos escrever: (ax—4).(3x+b)=a-3-x? +(a-b+3-4)-x+3-2 *. (ax-4).(3x+b) = 3ax? + (ab ~12)x— 4b. Os Segredos da Algebra para IME/ATA/OLIMPIADAS <> -—— —_ Problemas Propostos | ---- ——— questo 5.19 (CN-1954) a0 ee Decomponha 16x* — 1 em trés fatores, ee questao 5.20 (Harvard-MIT-2009) Determine 0 valor da soma 11? 1? +12? 22 43232), ag2 = Questo 5.21 (Turquia-2007-Modificada) Determine o valor de (100? 99?) (90? - 98?)...(32 2? (2-8) Questao 5.22 (OCM-1998-Modificada) Determine o valor de 1 — 2? +3? - 4? 4... 19982 + 19992. Questao 5.23 (Moscou 1945) Divida a 1?" por (a+b)(a? +b?)(a* +b*) (a +0). Questdo 5.24 (Moscou 1945) Oivida a — 2" por (a+b)(a? +b?)(a* +b4 A A {a pb }. Questo ao 5. 25 (Moscou 1946) rove que, depois de completar a multiplicagao e sarupay os fermos de (xs? ya, _ x99 + x100) , nao havera +2109). (cx? XP Mondmiog de grau impar. _ Cuesta g, 28 (CN-2008) Simy qd a2 " Piicando se, afrago a4 +b4 6a? onde a>b, obtém-se: a) a2 12» a* —b? + 2ab ae pe ‘ab b) a? —b? + 2ab Mazina d) a? +b? + 2ab 5 Produtos Notaveis 148 27 (AHSME-1951 1 CN-1998) Questao 5. 2 ~ x . ~ yr para XeR . Obtém-se- x4 - ~ 2 Simplificando a expresso /1 *| i 4 _ 2 c) X=* =a 2x Questao 5.28 Simplifique (a+b-c¢ +d)? +(a+b—c-d)* ~2|(c-a-b? - |. Questao 5.29 Simplifique (a +b)(a—b)(a? +ab+b?)(a? ~ab+b?)(a’? +a®b® +b”) Questao 5.30 (CN-1991) Simplificando a expressdo abaixo, para os valores a, b € c que nao anulamo denominador, obtém-se: (a? -b® ~c? -2be)(a+b—c) eres (a+b+c)(a? +0? ~2ac-b?) a1 b)2 c)3 d)a+bte e)a-bic Os Segrodos da Algebra para IMEATA/OLIMPIADA: S 149 ja, vimnos 08 produlos Notdveis de expoente dois, ou s| gor vereMOSs OS Produtos de expoente tide ae we Plevados ao te ae 85, 0u seja, elevados ao jain a gubo, varnos ll 641) cubo da Soma de Dois Termos. pata da soma de dors ermos 6 igual ao primero temo elevado rg voz0s.0 printer terme elevado ao quadrado vezes 0 se comme ais ros vere os ¢ undo termo elt ‘gundo termo, mais ;gundo termo elevado ao quadrado, mais »s 0 primeiro termo ve levado ao cubo 0 50 3 a +b) = a8 s 3a’ + dab? + b>] Demonstragao: a: bp-(a4 by (a +b) o (a+ by = (2? +2ab+b?)-(a +b) e (a+ b= a? + 2a% + ab? + a’ + 2ab?s b? «fa + by’ = a8 + 3a2b + 3ab? + b*). Exemplo Resolvido 170: Desenvolva (x +3)°. Resolugao: Podemos escrever: 3 (x43) <9 4.3.x? .343-x-3243% 2 (x43) = x9 49x? +27K+27. a Resolvido 174: Desenvalva (3x +2)°. *Solucao: Podemos escrever: (3x42) *+2)" =(3x)? 43-(3x)?.243-3x-22 +2° : 3 Gx 27 2749 5 54x24 36x48. 84 "4 Cut iy 3 eat diferenga de Dois Termos: ——— wcrengade Dois Termos: ___ iferenga entre dois termos @ igual ao primeiro terme elevado #6 Menos tre lem, mais es vezes 0 primeiro termo elevado ao quadrado vezes ° segundo Wathagg ten vezes 0 primeiro termo vezes 0 segundo termo elevado a0 Men °S © Segundo termo elevado ao cubo. [a — bp = a? 3a% + 3ab?- b? 5 Produtos Notaveis Demonstracao: fa vy -(a by (a-b) <> (a b)° (a 2ab +b?) (a pb)’ = a? 2a"b 1 ab? ab + 2ab? — p3 3a*b + 3al Exemplo Resolvido 172: Desenvolva (x 2p. Resolugao: Podemos escrever: (x2) = x8 432-243-427 429 2 (x2) = 62 a H2K-g Exemplo Resolvido 173: Desenvoiva (x — 2y). Resolugao: Podemos escrever: ay? (x-2y)> = x3 -3-x2 .2y +3-x-(2y)? ~(ay)? 3 y + (3x2) = x3 ~6x2y + 12xy? — By? 5.13) Cubo da Soma mais Cubo da Diferenga: es a an Quando somamos 0 cubo da soma com o cubo da diferenga, obtemos produto notavel interessante: o dobro do primeiro termo que multipica ¢ quadrado do primeira termo mais trés vezes 0 quadrado do segundo terme. (a +b)’ +(a — b)° = 2a(a? + 3b") Demonstracdo: 3 3 (a+b) +(a-b)? =a9 + 3a%b + 3ab? +b? 40? —3a2b~Sab?-b” 3 3 - 2 (a+b) +(a-b)’ = 209s ab? ~. [fa by’ +(a-b) =2ala’ +3 ] Exemplo Resolvido 174: Efetue (x +3)? + (x-3)° Resolucao: Podemos escrever: (x43) 4(x~3) =2x{x2 43.92) (x43)? +(x-3)? =ax(x? +27) (x+3) 3 +(%- 3) = 2x9 54x, Os Segredos da Algebra para IMEATA/OL IMPIADAS. 151 Exemplo Resolvido 17: Efetue (2x 15)” 1 (2x 5) cao: Podemos escrever Resolut : 5)° - 2-(2x)|(2xy? 43.5? | jaxs5)) (2% 3 2 4 axis) +(2x 5) 4x(4x? +75). (2x5)? «(2x 5)° 1042 300% 5,14) Cubo da Soma menos Cubo da Diferenga: 5 sublraimos 0 cubo da diferenga do cubo da soma, obtemos um produto ante: 0 dobro do segundo termo que mulliplica trés vezes © termo mais 0 quadrado do segundo termo. asp)? (aby? = 2b(3a? +?) Quando notavel intere: quadrado do primeiro a? +3a2b + 3ab? 1b? (a? -3a’b + Sab? b*) favoy (arb) (aby? —a® +3a% + 3ab? + b° a? + 3a’b- Sab? +b? (as by’ (a~b)” = 2b(3a” ‘| (arb)? -(a-b)? = 6a% +2b° 3 Exemplo Resolvido 176: Efetue (x +5)°~(*-5) Resolucdo: Podemos escrever: 3 > (x35)? (x5)? =2.6(3-x2 +5?) = (x45) (9) = 10(3x? +25) > (x+5) —(x—5)° = 30x? +250. 3 3 2, xemplo Resolvido 177: (3x + 7)? -(3x-7) = 378% 686. Re in Podemos escrever: K+7)P (ay -7) =2.7|3 (3x) P| * (847) (ax 79? — 44(27x? +49) * (3x47) *47)" ~ (3x7)? = 378x? + 686. 152 5 Produtos Notaveis 5.15) Identidade de Cauchy (Soma): Podemos escrever o cubo da soma de um modo mi ai cae identidade de Cauchy: (a + by ‘ aoa | a iy a? + bP + Sabla Demonstracao: =a2 +3a°b+ Sab? +b? |(a+b) 3 (a+b +b? + 3ab(a+5] 3 Exemplo Resolvido 178: Desenvolva (m+10) Resolugao: Podemos escrever: (m+ 10)° =m? +109 +3-m-10(m-+10) = (m+10)° =m? +1000 +30m(m +10). Exemplo Resolvido 179: Desenvolva (3n + 4y. Resolugao: Podemos escrever: (3n +4)? = (3n)? +43 +3-(3n)-4(3n +4) «. (3044)? =27n? +64 +36n(3n 4-4). 5.16) Identidade de Cauchy (Diferenga): Podemos escrever o cubo da diferenga de um modo mais cdmodo, € a chamada identidade de Cauchy: (ja — by = a? b?- 3an(a — b) Demonstracai 3 3 (a-b) 3a’b + 3ab2 — p3 (a —b)* =a® —b? —3ab(a-b)}- s Yeremes como expressar a soma de cubos e a diferenga de cubos. Poet fata a Patt das identidades de Cauchy, temos também @ . '@, que so chamados de Produtos de Warring. Os Segredos da Algebra para IME/TA/OLIMPiADAS 153 +b? - (a+b) -3ab(a +) a) soma de cubos: [a> + pemonstracao: ey eS =a +b? +3ab(a+b) » fa? + b (arb? 3ab(a +b) C8 +b)" 3ab(a +b) (a+b »)_Diferenga de cubos: > (ab)? -b)? + Baba bj Demonstracao: a ab) =a? -b® -3ab(a-b) ~. fa? b? (aby + dab(a_o) [a= BY =(a-by + 3ab(a by c) Soma de cubos (Warring): a? +b° = (a +b)(a” ~ab +b”) Demonstracao: a°+b®=(a+b)°—3ab(a+b) <> a®+b® =(a+b)|(a+by? Sab | © a+b? = (a+b) a? +2ab +b? -dab| [a «0° =(@-bj[a* ab -b*) a? bp? =(a- b)(2? +ab+b?)). 4) Diferenga de cubos (Warring): |@ Demonstracao: ; 2 #0 (ab)? +3ab(a—b) <2 a? -b? =(a-b)| (ab) +38 | & *emplo Resolvido 180: Efetue x? + 4°. Res ‘lugao: Podemos escrever: P44 =( a(x)? 12x08 4) 3 x4)? -3-x-4(x44) > x44 = 5 Produtos Notaveis 3 Exemplo Resolvido 181: Efetue (am)? — 33, Resolucdo: Podemos escrever: (amy? ~ 3° =(2m-3)° +3-(2m)-3(2m-3) = (amy - 3° =(2m- 3)° +18m(2m -3). Exemplo Resolvido 182: Efetue (5a)? +(2b)°. Resolusos Podemos escrever: (Sa)? +(2b)° = (8a +2b)| (Ba)° ~(5a)-(20) + (2)"| = (5a)’ + (2b)° = (5a +2b)(252 - 10ab + 4b?). Exemplo Resolvido 183: Efetue (11 xP -y. Resolugao: Podemos escrever: 3 (14x) -y? = (1-9) (11x)? +(14x) yy | 3 : > (11x) HP =(11x-y)(121x2 st 1 xy +"). Os Segredos da Algebra para IME/TA/OLIMPIADAS 155 = Problemas Propostos questa0 5.34 (Harvard-MIT-2007)_ = i nlimeros reais tais que x-y—4 @ x° y? 28. Detwmine 2 quest4o 5,32 (CN-2006)_ x(x? +x-y)+y?(y+4) LX’ yxy 40, obtém-se: -se a fragdo simplificando-se a "a0 Zaye ayx-y+t b) x-y-1 ©) x+y-1 ay exr t-E+7 Questao 5.33 (CN-1980) 2x? - 4x +8)(x? -4) simplificando Eee a) v2(x+2) b) 2(x-2) ©) 2 (x? -4) ae e) 2 , vamos encontrar: Questo 6.34 (CN-1983) 2. 2 2_y2 xara’ * axve) x y ) , vamos encontrar: a + 3x2y + 3xy* +y> oc) x+y Simplificando a fragao 3) z(x+y) b) z(x-y) ay e)Z+y Questao 5.35 (a+b)? —4(a? +b?) Sinpifique ——__—_. (2 +b) -(2? +b?) 156 5 Produtos Notavei, Questo 5.36 (AHSME-1952) (x+1) (est (= xt ‘fe 1? (x? iff (ot | (84h | EOS yo b (8 +9 on alleieeyP «|e 47 Questao 5.37 Determine a? — ab - ab? +b°. Questao 5.38 (Harvard/MIT-2000) Calcule 2000° —- 1999. 20007 — 1999? - 2000 + 1999° . Questao 5.39 (Harvard/MIT-2007) 2 3 3 3 3 Calcule 2—1, 30-1 47-1 5-1 6 a1 caue 2 a1 3-1 49-1 169-41 241 3841 444 16847 Os Segredos da Algebra para 'ME/ITA/OLIMPIADAS 157 jveremos agora & quarta poténcia de dois termos e tancias de temos reciprocos. Autilizaremos para esenvO" ae conver pol ta Poténcia ¢ da Soma: tancia da soma é dada por —_ Aaquarta po (a + byt = at + 4a% + Ga%b? 4 gan? ps pemonstracao: ja-b}t (a+b)? (a+b) ° (a+by* = (a°+3a% +320? +b9)(a -p) 5.17) Quet (a+b) -a+ 3a°b + 3a’? + ab?+ ab + 3a’? 4 gab? s bt [a b)' = a*+ 4a°b + 6a*b? + dab? + p* Exemplo Resolvido 184: Desenvolva (x+ 2y . Resolugao: Podemos escrever: 4 8 (xs2) =x 44-x8.246-x2-27 4.x. 23 424 (x+2)* =x4 48x? + 24x? +32x +16. Exemplo Resolvido 188: Desenvolva (2x +3)". Resolugao: Podemos escrever: ax 4x2 9 27 (2+) (2x) 4. (2x)° 34 6 (2x)? -3? +4-(2x):3? 434 sin ad (2+3)* < 46x4 + 96x? 4 216x2 + 216x +81. 43 3a +3ab? -v?)(a-P) Ao b) = (a-b)* =( ee UU ee 7 5 Produtos Notaveis - 3a%b + 3a’b?~ ab? a% , 3ary2 Sap? +bht Exemplo Resolvido 186: Desevolva (x-1)*. Resolugao: Podemos escrever: (xt) x4 do 1462? Pd xP tf = (xt) =x 4x? + 6x? 4x41. Exemplo Resolvido 187: Desenvoiva (4m-3)" Resolugao: Podemos escrever: 64m? 16m? 27 (4m_—3)* = (4m)* ~4.(4m)?.3.+6.(4m)? -3? -4.(4m).3° +34 «. (4m-3)! = 256m4 - 768m? + 864m? — 432m+81. 5.19) Identidade de Legendre (Soma): ‘Somando as duas identidades anteriores, chegamos a identidade da soma de Legendre para a quarta poténcia. 2 (a + by’ + (a — b)* = 2(a? + b?) + 80%" Demonstragao: t 3 +b (a+b)! =(a—b)* (axe ay)! +(3x-4y)" 2(9x2 + 16y2)°4.115202y2 4 5 (axe dy) # (3x49)! = 2(8 04 + 288x2y? + 25694) 4115202)? 4 4 = (axr4yy +(8x-4¥) = 162x4 + 576x?y? + 512y4 +4152x2y? (ax+4y)* +(x —4y)* =162x* +1728xy? + 512y4. 5,20) Identidade de Legendre (Diferenga): ubtraindo as duas identidades anteriores, chegamos 4 identidade da diferenga de Legendre para @ quarta poténcia : ee (a +b) - (a - b) = Bab(a? +b?) Demonstracao: (a+b) -(a-b)* - a4 + 4a%b +627? + dab? +b — -(a* — 4a%b + 6a°b? - 4ab? +b4) =a? + 4a%b +6ab? + dab? +b4 -a* +4a°b — —6a°b? + dab? +b* a? +b? )}. (a+by* -(a-b)* (a+b)' -(aby* =a +8ab3 ~ [(a+b)*-(@-»)' = Bab/ Exemplo Resolvido 189: Efetue (7x +2y)" -(7*- ay). Re - nusaee Podemos escrever: (x 5 *2r)!-(7x-2y)* =8 (7x)-(2y)| (7)? +217 | (7 4 + 2y)* -(7x-2y)* = 112¥y (49x? +4y")- em, 4 Plo Resolvido 190: Efetue (14m+5n) -(11™- 5n) 400 5 Produtos Notaveis Resolucdo: Podemos escrever: (11m+5n)* (11m-Sn)* =8-(141m) (50) (11m? “(ang = (t1m+5n)* -(14m—5n)* = 440mn(121m? + 26n2). 5.21) Quinta Poténcia da Soma ‘Aquinta jinta poténcia da soma é dada por ee (a + by) = a&+ Sa4b + 108°? + 100%? 4 Sab +] Demonstraca: (a+b)’ =(a+b)* (a+b) (a+b) =(a* +4a°b+6a?b? + 4ab® +b4).(a+b) SS (a+b)? =a° + 4atb + 6a5b? + 4a7b? + ab* +a4b + 4a%b? + 6a2b? + dab! ats -. (a +b)? = a + Sa4d + 10a%b? + 10a%b? + Sab4 + 6°], Exemplo Resolvido 191: Desenvolva (x+1)°. Resolugao: Podemos escrever: (x+4)° = x5 45-4 1410-03 12 4 10-x? B45 x the = (x+1P = x9 x4 4 10x34 10x? + 5x +1, 5.22) Vamos desenvolver a quinta poténcia da diferenga: __—— Aquinta poténcia | oa diferenca é dada por <7 Sh a ee 5 Demonstracao: (a-b)’ =(a-b)*-(a-b) (a-b)? = (a4 ~ 4a%b + 6a%b? — abi +b*).(a—b) 23 (a-b)° = a8 — day + 6a%b? — 4a7b? +. ab* —aty + 4a%? 62°” * ; 4 4ab* -P 5 * l(a — by? = a®— Sab + 100%? — 10° + Sab*— b Os Segredos da Algebra para IME/ITA/OLIMPIADAS 161 exemple Resolvido 192: Desenvolva (x—2)°. Resolugao: Podemos escrever: 4 8 16 5 x5 5.x. 2410-x9-2? ~10.x?.23 45.x.24 28 Asoma dos dois produtos notaveis anteriores é dada por (fa +b) +(a — bp = 2a(a* + 100%? + set) 6.23) ASoma das Quintas Poténcias da Soma e da Diferenga: Demonstragao: (a+b) + (a-b)® = a + Baty + 100%? + J0065 + 5ab4 + DE + a5 — — Bake + 100%? — 1006 + Sab4 - DK (a +b) + (a — bd)? = 2a° + 20a%? + 10ab4 la +b) +(a — by = 2a(at + 10a*b? + 6b*)). 5 5 Exemplo Resolvido 193: Determine (3x+1)’ +(3x-1)’. Resolugao: Podemos escrever: (8x4)° + (9x —1)8 =2-(8x)_| (3x)? + 10-(Bx)° P +5-7* | 9x2 % (8x44)? (8x _1)° -6x(atx4 +90x? +5) © (xs 4P + (3x ~1)° = 486x° + 540x° + 30x. Exe 5 5 oe Resolvido 194: Determine (2x+5) *(2* -5). “G80: Podemos escrever: (2 4.5)5 | 2 4 (24° 2(2n)| (ae) +40: (2xh 8655 4x’ 5 . Produtos Notaveis = (2x15) +(2e-8)° = 4x(16x4 + 1000x? +3125) (2x15) + (2x-8)° - 64x° + 4000x° + 12500x 5.24) A Diferenga das Quintas Poténcias da Soma e da Diferenca, A diferenga entre os dois produtos notaveis anteriores é dada por — a+b) + (a- bj? = 2b(5a* + 108%? ') Demonstragao: (asby’— (a-b)>= gf +5a%b + Tre +10a7b? + Bast +8 (# 5a4b + 1032 ~10a%? + Bas 1) (a +b)’ - (aby? = 10a‘ + 20a2b? + 2b° fas by + (aby? = 2b(5a4 + 100%? + b*} . 5. 5 Exemplo Resolvido 195: Determine (2m+3) +(2m-3)'. Resolugao: Podemos escrever: | (2m +3) +(2m-3)5 = 2-3. 5.(2m)* +10-(2m)?-3? +3* Deis coal l 16m4 4m? = (2m+3)° +(2m—3)° = 6(80m4 + 360m? +81) 2. (2m+3)° +(2m—3)° = 480m4 +2160m? +486. Exemplo Resolvido 196: Determine (t+1)° +(t-1)”. Resolugao: Podemos escrever: 5 (t+4P 4(t-1 =2 1-(5.14 +10-12 Pt) = (tea? 4(t ay = -{5t* +101? + 1) 5 » (Leap +(t-4 = 1014 4 2012 42. Os Segredos da Algebra para IME/ITA/OLIMPIADAS 163 ‘Problemas Propostos ueet0 4 oo 2. p2 =k e aD-X. Determine (a+b)* @ (a—by pado questo 6.42 _ gejam x e y numeros reais positivos satisfazendo Riyatextiyt 18 Determine XY - uestéo 5.43 _ a 2 sy? 2 Be 8 8 i x x K+ - Dados x e y reais com —> o + —s =k, determine % Y + y em xy ety xeay® Bay termos de k. Questo 5.44 (IMO-Longlist-1992-Adaptada) 1 5125 | Simplifique Pr Questao 5.45 _ — Sejam a e b nimeros reais no nulos tais que xe y satisfazem o sistema | ax +by=2 ax? + by” = ax? +by? = 56 ax? +by* = 272 D étermine o valor de ax® +by® Ques es SS (eue session M2 -MiT-2009) et mine ax? +by> se og numeros reais a, b, x @ y satisfazem aS equagées | by | V3, ax? by? 7, ax? + by? = 16, ax! roy’ =42. 164 5 Produtos Notaveis Questao 5.47 (OBM XXXI - 22 Fase - Nivel 2) Determine ax? + by? se os nuimeros reais a, b, x e y satisfaze, m as Uap jee ax «by =1, ax” +by” =2, ax? +by? =5, ax’ +by4 -6. Questdo 5.48 (AMC-2007) Suponha que o numero a satisfaga a equagdo a+a'=4. Qual ow . © Valor ¢ at+a‘? a) 169 b) 172 c) 192 d) 194 e) 212 Os Segredos da Algebra para IMEATA/OLIMPIADAS 6 166 desenvolver a soma di ci A nos agora s de termos reciproct j ven pciprocos x"), dada a x! 1 ox tok, com k>2. Primeirar , condiga a Mente iremos ver algumas regras praticas, a0 final veremos sua generalizagao. a de Termos Reciprocos: os quadrados de termos reciprocos, dada a sua soma ados de dois termos reciprocos . 5,25) Som a) Soma di ‘Asoma dos quadra 2,1 ,2 x4 k Mz Demonstracao: 1 1y xe-=k © [x = x x 1 1 3] 2 2 2 2 ea P24 —Z eke [Xe + =k*-2). x peek | b) Asoma dos cubos de termos reciprocos, dada a sua soma. ‘A soma dos cubos de termos reciprocos € dada por =i? oo B2x(1)s(4) Li) Demonstragao: 1 wteko (xo ©4343 1). x 2 843k 4 x : Asoma das quartas poténcias de termos reciprocos, dada @ sua ‘oma, A . Soma das quartas poténcias de termos reciprocos 6 dada por __“_"S~=_s#K“—_¥_—sWMK“——ht a 5 Produtos Notaveis d) Asoma das quintas poténcias de termos reciprocos, dada a sua soma. Asoma das quintas poténcias de termos reciprocos é dada por xo 4 L = k5 5k? + 5k x Demonstracao: 5 7 xttek > (+2) =k x x) “4 = 8 45x4 | 1) 40x (+ x} x \ ° +50 +10«+10,(7) x eo 845(x241] 10 1 % +g] H10[ x4 1 5 e x8 +8(k2 ~ak) 10k +L Wo co x8 48k9 — 15 410K +5 =F x ee x8 25k ke ate fx pL ag8 51? 45K. a A soma das sextas poténcias de termos reciprocos, dada 4 sua oma das sextas poténcias de termos reciprocos € dada por 1 x° += =k 6k4 49k? - 2 Os Segredos da Algebra para IME/ITA/OLIMPIADAS 167 pemonstra dO: 5 6 6 1y x84 ext +20+15x2+15,.8 1 px! x 6 2{ 1 4 _ 442 42) 5 WB an +57 8K -4k +2)+15(k? 2), 20 1 > wo rb rg = 24k? +12 +15k? 30 +20 1 k8 —6k4 49k? -2. 1 4 2 6 5 8 ero -9k2 +2 => x Observacgdo: Com um pouco de manipulagdo algébrica, podemos escrever a soma das sextas poténcias da seguinte forma: 18+ = (KP 2) ae? 1). Generalizando por equacao do 2° grau e lembrando que k22. Podemos generalizar a soma da enésima poténcia resolvendo a equagao do segundo grau formada, assim sua solugao geral é dada por: Demonstracao: 1 2 tosk o xeatekx oo x2—kxt1=0, Aa(-k) -4-44 —(-k) sak 4 24 -A=-420; x= , entéo: Se K+ vke~4 Vea 2 5 Produtos Notaveis 168 se x —KoNK'=4 , ent&o 2 Observacao: Os primeiros desenvolvimentos s4o formas Praticas fica a cargo do leitor decora-las ou nao. S © rapidas Vamos agora desenvolver a soma de termos reciprocos x", 1 dad, xn aa 1 a condigao x-—=k, com k>2. Primeiramente iremos ver algumas regis x 8 praticas, ao final veremos sua generalizagao. 5.26) Diferenga de Termos Reciprocos: ‘a) Soma dos quadrados de termos reciprocos, dada a sua diferenca. A soma dos quadrados de dois termos reciprocos € dada por k2 42 Demonstragao: ; / a. b) A diferenga dos cubos de termos reciprocos, dada a Sua aiferens A diferenca dos cubos de termos reciprocos é dada por Demonstraca: i 1 xX-s=k © | x-- ae x 0s Segredos da Algebra para IME/ITA/OLIMPIADAS 169 yAsoma das quartas poténcias de termos reciprocos, dada asua diferen¢a- ‘asoma das quartas poténcias de termos reciprocos é dada por 7 ; xa 4 ak? 42] ) A ay? 1) o x -4x vet + 2) ‘I x’ 1 o xt -a{k? +2) +6435 ib x8 42-8 6 x x* xt 4k? 2+ k4 44k? +2 4) A diferenga das quintas poténcias de termos reciprocos, dada a sua diferenga. Adiferenga das quintas poténcias de term: 5 _ 1 48 5k? + 5k os reciprocos € dada por x x Demonstracao: 1 wok = (x-4 oe 1 rare 3 aye (4 5 #544 (2) .1048-(4) -1092-(2) wse(2) (3 x x x x x —SEo_——(—————————— 5 Produtos Notaéveigs 170 1 eo 5{k° + 3k) + 10k Lake eo x9 5k? — 15k + 10k x es x8 5k? 5k 5 4 5k3 4 5k]. e)A soma das sextas poténcias de termos reciprocos, dada a uy diferenga. ; Asoma das sextas poténcias x8 Lk? + Ok + Ok? +2 de termos reciprocos é dada por Demonstragao: 6 [x2] = x8 6x4 1512-20475 x > Kona of xt +h) e19f2+5)- 20 x = K-84 ~6{k# +44? +2) +15(k? +2)- o> ko = x8 + Lok 24k? 12+ 15k? +3020 x 1 => k® = x® + 6k4 9k? 2 x84 =K8 4 6k4 4 9K? +2. x x Observagao: Com um pouco de manipulagdo, podemos escrever: 6,1 k? 42)(k4 4k? +1). Generalizando por equagao di 2. nco'por | jo 2° grau e lembrando que ke a0 mi << ges do que K28- od Podemos generalizar a soma da enésima poténcia resolvendo 4 equa segundo grau formada, assim sua solucao geral é dada por: then 4 2 Os Segredos da Algebra para IME/ITA/OLIMPIADAS pemonstiacdo: Ke XP ATEKK PRK 120; A =(- Pg 1(-1) Ke —(-k) vk 4 x 24 m1 nek? +420; x= ‘Aqui temos que considerar dois casos: a para n impar. kevk’ +4 entdo: 2 vf n 4 kavk2 44 -{ 2% kaka) | se x= Para n par: Se a Ke vk? 4 2 , entao: ta _ ~~ 5 Produtos Notaveis 172 1 : Sabendo que x +— —16, qual 7: xte q! © Valor de 42 Exemplo Resolvido 19) » x 0: Podemos escrever: 1 Resoluca = 256-2 2,1 462-2 > x? + x 1 Exemplo Resolvido 198: Sabendo que X + x 10, qual o valor de 3 1 > x Resolugdo: Podemos escrever: 1 100-30 .. 4 12109-3410 = P+ x Exemplo Resolvido 199: Sabendo que x + 1 _6, determine o valor de x tet x Resolugdo: Podemos escrever: 1 x8 644.6742 > x8 41-1206 4.36 +2 x x =1298-144 .. Exemplo Resolvido 200: Sabendo que x+i-5 determine o valor 0 x Oy 1 x Resolucao: Podemos. escrever: 5 1 4-55 _5 53 xe §-5°4+5.5 8 +4 3125-6254 25 Os Segredos da Algebra para IMEATA/OLIMPiaDAS 173 Resolvido 201: Sabendo que , , 1 x exemplo 3. determine o valor de a, +. x mo pesoluGao: Podemos escrever: 4 = 6 ooh a9? 6-54 49:97 -2 = x8 +2 -729-6.6149.9-2 x oy x84 = 729-486 + 81-2 exemplo Resolvido 202: Se x , 72, qualo valor de x10 x Resolugao: Podemos escrever: f 100 10,1 _{ 2422-4 2° esx > B=] S| +} x 2 224) \100 , 5,100 +(2) = B= 1100, 1100 2) Agora veremos as identidades para trés variaveis, identidades muito tteis no desenvolvimento das relagdes de Girard (polindmios) e também em fatoragdes. 5.27) Quadrado da Soma de Trés Termos: O quacrado da soma de trés termos é igual ao quadrado de cada um dos trés ‘emos mais 0 dobro do produtos tomados dois a dois. (a +b +c)? = a2+ b?+ c+ 2(ab + ac + be) Demonstracao: @sb4 of =(a+bec)(a+b+c) o ( a 2 Bebe cP. ay ab + ac + ab + b?+bo+ac+bo+e ° {8+ +c) = a24 p24 c2+ Zab + 2ac + 2b¢ oo S +b +c) = 224 p24 c2+ 2(ab + ac + be) 5 Produtos Notaveis 174 Consequéncias do quadrado da soma de trés termo: a) O segundo termo €@ negativo: ——— )* 2 ry 2 fa — b + of = a + bP + CF + Bac — ( ab ~ be) Demonstragao: fa bac) = (@-b4ep(a~b +e) s (a b + c)’= a?- ab + ac ab + b?— be + ac ~ be y ej (ab 4 e)'= a4 b? + 67 ~ Zab + 2ac ~ 2be fa b+ op are b?+c%+ 2(ac ab ~ be) b) Oterceiro termo é negativo: (a+b -cf= Demonstracdo: fa+b-cf=(atb-c)(atb-e) © (a +b —c)?= a%+ ab - ac + ab +b? be - ac — bo +o @ (a+b- of = a+ b?+ c+ 2ab - 2ac - 2be (a + b - c)’= a?+ b?+ c? + 2(ab ~ ac - be} c) O segundo e o terceiro termos sao negativos: (a — b — cj’ = a2+ b?+ o% + 2(be — ab — aC) Demonstracao: (ab cf =(a~b~—c)(a~b~c) 2 (a-b-—o)?-a? ab — ac ~ ab + b2+ bo — ac + bo + 2 (a - b~ c) =a?+ b?4 c?~ 2ab — 2ac + 2be +b? + 0% + 2(be - ab — ac)]. 2 + b?+ c%+ 2(ab — ac — be) 2 Os Segredos da Algebra para IME/TA/OLIMPIADAS 175 exemplo Resolvido 203: Desenvolva (a+ 2b +30)? pesolugao: PO ‘ ; p+ 3c)? = a + (2D)" + (3C" +2[(a-2d)+(a.30) (ap 3c)] demos escrever: (aed . (a 420+ 3c)? =a? + 4b + 9c? + 2(2ab + 3ac + bbe) (a+20+ 3c)’ =a? + 4b? + 9c? + dab + 6ac4 12b¢ Exemplo Resolvido 204: Desenvolva (4—2m+n)* Resolugao: Podemos escrever: (d-anen)? <4? +(2m)? +n? + 2[ 4n— (4.2m) (2m-n)] = (4-2m+n)? = 16 + 4m? +n? + 2(4n-8m —2mn) (4-2m+n)? =16 + 4m? +n? + 8n-16m—4mn Exemplo Resolvido 205: Desenvolva (x +y— 2z)* . Resolugdo: Podemos escrever: (x+y-2z)? = x? +y? + (2z)? +2(xy -x-22-y-2z) = (x+y 22) =x? sy? +42? +2(xy- 2xz-2yz) (x+y—22)? =x? +y? 4422 + 2xy —4xz—4yz. Exemplo Resolvido 206: Desenvolva (1-x-y)°. Resolugao: Podemos escrever: (oxmyP Rn? by? 4 2(x-y—tx—t-y) > (tox -y)? ata x2 yy? +2(xy-x-y) © (oxy? = ta x2 4 y2 s Oxy 2x -2y $28) Wdentidade de Lagrange para Trés Termos: entidade de Lagrange para tés termos € util em fatoragdes mais Febus, “202s, cuja percepgéio do leitor esteja bem apurada. 5 Produtos Notaveis 176 Demonstracao: | 2 E=(ax+by+ ez) Pee! + (az— ox)" + (bz -oy)? 5 E=(ax)’ +(by)” P+ Darby + an-CE 1 * Bre +( (ay)? Preyer (bx? a ae *(02) Pera > 2. Zz: 5 Ea atx? + by? 027? +a2y? + bx? + atzt 22? + 02x? 4 p27 bez? «2a = a — — 5 ea at(x2s y2 +22) 1b?(y? rx? +2 2) 46? (2? 4x? 1y?) 2 2 [ax -by +ez)" +(ay bx)* + (az—cx)° + (bz -cy) =(2? +b? so) ED 2 2 Exemplo 207: Desenvolva (x +2y+3z) +(y-2x) +(z~3x)? + (22 af Resolugao: Podemos escrever: 2 (+ 2y #32)? +(y 2x)? + (2-3x)* + (22-y)? = (P42? 43?) x2 sy? 2) (+ 2y +32)? +(y -2x)?+(2-Bx)? + (22 By)? = 14x? +y? 42). 5.29) Produto Dois a Dois Elevado ao Quadrado: Podemos também desenvolver a soma dos produtos dois a dois elevato 20 quadrado, segue o mesmo raciocinio da soma de trés termos: [ab + be + ac)*= (ab)*+ (be)*+ (ac)* + 2abc(a + b +o) Demonstracdo: E-(ab+be+ac)* => E=(ab+be+ac)-(ab+be+ac) = E=a%p? + ab’c + a%bc + abc +b2c? +abc? + a2bc + abc? +207 (ab + be + ac)? ~a?b? + p2c? +a%o? + 2a%e +2ab7c + 2abe” + |fab + be + ac)? (ab)? + (be)? + (ac)? + 2abe(a +b +c)}- Exemplo Resolvido 208: Desenvolva (2y + yz+ 2zy. Resolugao: Podemos escrever: 2 (2y + ya+2z)? =(2y)? + (yz)? + (22)? +2-2-ye(2+y +2) 2 > (2y+yz422) = Ay? +2? +42? 4 ay2(2+y +2). Os Segredos da Algebra para IME/ITA/OLIMPIADAS 477 xemplo Resolvido 209: Desenvolva (m+ 4mn 4 4n)? Resolugao: Podemos escrever: imem-4n+ 4n)? =(t-m)? =(m-4n)? +(1-4n)2 42-4. 4n(t4m4+4n) = (m= 4mn~4n)? =m 2 + 46m?2n? + 16n? + Bmn(1+m+4n) = Problemas Propostos Qquestdo 5.49 (AHSME-1952-1954 / CN-1986) ge yet) =3.entéo x° + é igual a: a b)2 <)0 a3 e)6 Questo 5.50 (CN-2014) Seja x um numero real, tal que x+2 9. Um possivel valor de x2 6 a. x x Sendo assim, a soma dos algarismos de “a” sera: a)11 b) 12 c) 13 d) 14 e)15 Questo 5.54 . 1 3.1 Sea x um numero real ndo nulo tal que X+>=4, S© * toy x. m+n =z =N, determine o valor de —— - x m-n Suestio 5.52 (AMC-2007) Suponhe que o numero a satisfaga a equagdo 4=a+@ sate : e b) 172 c) 192 e) 212 “1 Qual o valor de af S_ ~§ Sn 5 Produtos Notaveis 178 questao 5.53 (Stanford-2 2010) Ss ; a se x° 1 _7, determine o valor de a tS — x gues 5.54 — —_—— a Sy Se et =1, determine 0 valor de 5x Questao 5.55 (Singapura) se x2 -4x+1=0, determine 0 valor de Gueste® 5.56 1 ge x+4= V2, determine o valor de x74 x Questdo 5.57 Seja r um numero real, tal que + ¥ 3. Calcule o valor de r> +: Questao 5.58 Seja r um numero real positivo, tal que Questao 5.59 (CN-1984) $624.2424%4¥ 42 x y Z yz xm yw a) 192 b) 48 c) 32 d) 108 e) 96 8 : . acy e x+y +z=16 , 0 produto xyz €: Questao 5.60 (CN-1999) jd ee he Se m=n=p=6, mnp=2emnvmp rnp 11, podemes dizer que V9" mon p +— + — 6: np mp mn a)1 b)3 °)7 «18 22 Os Segredos da Algebra para IME/ITA/OLIMPIADAS 1479 uestao 6.5 5.61(CN-2011) = a,b e cnumeros reais, tais que — + sejam ab 25249 per? —9p ye a ©) p'r?-9 é pir? -10 e) p'r?—12p 4r yest 5 2 (Harvard/NIT-2008) as raizes da equacao x? 9x? +8x+2=0 s40p, q,r. Calcule Questdo 5.63 (AHSME-1981) Para todo nmero positivo x, y ez, 0 produto ptyez) (x tay t42 "Oy tyztxz)"[Oy) zy" +02)" | 6 igual a: b) x? +y? +22 c}(x+y+z) a) xyz e—1 XY +YZ+XZ Questo 5.64 (AHSME- 1991) Se x. f2 —1__ ~ 1 = 20, ent&o x? Vet za x2 4x41 1 9505 by 20 c) 51,005 d) 61,25 e) 400 6 igual a: Quests Auestao 5.65 Cae o valor de (4) (4) z 180 Questao 5.66 Determine o valor de (2 S Questdo 5.67 (CN-1998) 1997 1997 2 Sejam x (ene 5), oy 2k valor de 4x2 3y? é& a)1 b)2 c)3 d)4 B) —— 5 Produtos Notaveis aye” ae 10 Os Segredos da Algebra para IME/TA/OL pgp) ns S 181 30) Iden! idades de Argand: oe entidade que requer uma boa criatividage aes Outta id 'vidade do leitores a ideniid, argands também encontrada em olimpiadas de Matematios = lentidade de 5, Aidentidade de Argand € dada por: ica € em colégios militar (a? + ab + b? (a? ~ ab. b= a + arb? bf emonstracao: 2\fo2 — 2 A 2,2 fra De) ab +0?) a ab FBP bea? sath? at? G tab + b (a? —ab+ b?)= ats a? pl, +b* Consequéncia para b = 1: £E59e caso particular da identidade de Argand é bastante Gtk (@ hatt P)(a? -at+ *)= at + a2. + 44 oat? ar dewey, Generalizando a Identidade de Argand, temos: Podemos. generalizar a identidade de Argand para a soma de uma poténcia qualquer, assim temos: (2 ab" +b™)(a2—a-b” +b™ = af™ 4 22M. p2ny pin Demonstracao: (Pm sam bP. ?)(a2m a bh bm) at” - pBPT + AMD + BPO Pryde. Daye 4 o2mH2" De bv’ [ea wpe )(aem vam pe 10) = a ae De b*]. § 2 has Resolvido 210: Efetue (40? +2pq+d7) (49 —2pq+q ). \e§ 2 { sao: Podemos escrever: (pa 4 2 4 2 o2sq! a+? )(4p? - 2pq +a?) = (2p) +(20) “4 7 4 ‘pq 4) 492 —20q +42) = 160" + 40%¢? +4 : — 482 5 Produtos Notaveig Exemplo Resolvido 241; Efetue (4917 + 7t+ )(491 7104) Resolucao: Podemos escrever: ‘ 4s at (ast? + 7e-A}(49° teat) = (Tt) (TY +1 = (ast? +Tt+ a}(49t? 7t+1)= 2401 tt 490? 41. Exemplo Resolvido 212: Efetue [ow How)” Be!" (32)? | ov)" Oy)” Gey" +32)" | Resolucao: Podemos escrever: E =| (xyP™ + (xy) -(32!" +(3z)"| os B= (ay) + (xy. (82)"" + (32) Am 4 xemy2m gen 22 434nz4n (xy) ~(xy)™ (Bz)! +(32)°") an B= xi™y Agora veremos a identidade para quatro variaveis ao quadrado, ou seja, 0 quadrado da soma de quatro termos. 5.31) Quadrado da Soma de Quatro Termos: _ © quadrado de quatro termos é igual 4 soma dos quadrados de cada termo mais 0 dobro dos produtos tomados dois a dois. [a-bscsd)? a? +b? 402 1d? +2(ab+ac+ad+be+bd+ od) Demonstragéo: (asb+crd? =(a+b+erd-(a+b+e+d) (avb+01d)? 22 vab+acvad+absb?+bc+bd+ac+ bere + ped+ad+bd red (arbsc+d)® =a? +b? +c? +e2+2ab + 2ac + 2ad + 2bc + 2bd~ 204 ia JP =a? +b? +02 +d? 4 2(ab+ac +ad+ be + bd+°4) . (atb+c+d Exemplo Resolvido 213: Desenvolva (x + 2y +2241)" . Os Segredos da Algebra para IME/TAIOLIMPIADAS 183 Resolucao: Podemos escrever: xeayr2eeP =? + (2yP 4(22)? vale BY) (22) + (2-1) + (2y27)4 (2y.1) 4(22. 4) (xeays2ee ty =x? + Ay? +42? ST 2(2xy 4 2xz 4x ays + 2y 427) o caso seguinte € @ soma de trés termos elevado @0 cubo, ou seja 3 t ‘I (a+ b +c), vamos desenvolvé-lo! 5,32) Cubo da Soma de Trés Termos: Ocubo da soma de trés termos 6 dado por. ~ 5 [axb-e) =a? +b? +0? +3ab? + 3ac? + 3a’b +3a%e + 3b2c + 3bo? + Babe Demonstracao: (a+b+c)° =(a+b+c)* -(a+b+c) © (arb+c)? =(a? +b? +0? + 2ab +-2ac + 2be).(a+b +c) @(a+b+c)’ =a3 +ab? +20? + 2a7b + 2a7e + 2abe +47 +b? + bo? + + 2ab? + 2abe+2b2e + a2c +b2c-+ 6? + 2abe + 2ac* 1 2bc* a eater eee a+b +0)? =a? sb?) 6? + Sab? + a0? 4 3a%b + 3a°c 4.36% + 3bc7 + Babe Podemos escrever essa soma de outras quatro forn formas, 2 ase saber: 7 satbadabiedasitis a) Forma de soma dois a dois [a+b +0)° (a+b+0+ 4) -a9 +b? +09 +d? +3a2b + 3ar%e + 3a2d + Sab? + 3b +3ac? + 3bc? + 302d + Sad? + 3bd? + 3cd? + abe + abd + acd + 6bed + 347°" 3abc + 3abd ~3abd : 3acd - 3acd + 3bed — 3bed 25 430d > 3 © (atbte+d) =a +b? +09 409 +3ab(a+b+c+d)+ + 8ac(a+b+e+d)+3ad(a+b+c+d)+3bc(at+b+ce+d)* +Sbd(a+b+c+d) + 3ed(a+b +6 +d)-3abe -3abd- 3acd ~3bcd (asb+o+d) =a? +53 08 +a 3ab(e+d)— aed +3 (a+b) + a+b+c+d)-(ab+ac+ad+be +bd+cd) Os Segredos da Algebra para IME/ITA/OLIMPIADAS 187 gora a quarta poténcia de trés termos! Observe como pod como podemos veremos ag0re 4 notara quando estiver na parte de identidades condicionais. cionais, | escreve-la, VOC vamos lal ) Quarta Poténcia de Trés Termos: | | 5.34) " | Xguarta poiencia de trés termos 6 dada por: —— | tad | (arb +e)! -at ebt vet 4(arb +e)’ (ab +ac+be) | 2(ab+ac+be)” 2abe(a +b +e) Demonstragao: 2 apvof -[@rseP] = (arb+c)! =| (2 +b?4 c2)+2(ab+ae-+be)] = (avb+c)*= a =b* +c# + 2(ab)? + 2(ac) + 2(be)? + +s{ab+ ac-+rbe)? +4] (a b+e) ~2(ab +ae +be) ab +ac-+be) 4 bt set 4 2(ab)? +2(ac}* +2(be)? - > (arbic)* - Blab ac-+be)? +4(a+b+c)¥ (ab+ac-+be) +4(ab +a¢-+be)” > (asdse)f at +b4 +4 + 2(ab)? +2(a0)* + 2(be)° - -4(ab +ac +bc) +4(a+ b+c)*(ab+ac +be) > (arboc)* af abt ve4 + 2(ab)? + 2(ae)” +2(be)° + “lab +¢)*(ab + ac +b) -4| (ab)? (ac)? (be)? +2abo(a r+ °)| | > lesbecjt at 458 sot —2fab)? -2(ae)? - 2(b0)° . -gabo(a 1b 1c) +4(a+b-+e) (abe ac Pe) “(sb 46)8 a4 bt pct a(asb ac) (ab+ac + PC) ~2abo(a +b +.¢)~2| (ab)? +(ac)’ + (bc)? + 2abe(a +P re) asp. 4 SY =at spt ict ya(a+ b+) (ab + ac + Po) ; (ab ag bat? C+ be)" - 2abe(a+b +c) 5 Produtos Notaveigs — — Problemas Propostos | Questao 5.68 (Imo-Longlist-1988 1 AHSME-1975) | sep, qe sao as raizes distintas da equagao x2 4y 20g | | + Nag | prig? +r é igual a a)-1 b)1 c)3 d)4 e)5 Questao 5.69 (Putnam -1939- Modificada) ‘As raizes de °° ax? +bx+c=0 Sao a, pey. Determine a 48 4 Questao | 5.70 | (AIME- -2008) etas trés raizes da equacao 8x3 +1001x + 2008 = 0 Determine | ‘Sejam f, 2 (rss) +(s+t) a (tery. Questao 5.71 (Stanford-2007) Seriset-3, +s +t? =ter+s 34.43 =3, calcule r-s-t. Questao 5.72 (Stanford-2007) As raizes de x2 -7x2 -6x+5-0 sao a, bec. Calcule (a+ b)(a+c)(b-¢) ae (24 +2? 1 1)(44 44? + 1)(6* +67 +1)...(324 +3z+t) (f+?+ 1)(3* +32 +1)(5* +5 +4)...(30 zat determine o valor de A-1053. Questao 5.73 Sabendo que A = Questao 5.74 i 4) Woste ue (ye = Gy)? ra) = roamcer vie" cht Q Questo 5.75 (Noruega-1996-Modi jcada) xyz que ‘Sejam x ye y € Z numeros naturais com x < y af + abt — 2\2 +202)" app at 4p¢ — (a2 + 2b? + 2ab)(a? » 2b? 2ab) Consequéncias: _ a) Parab=1 ; — ate att = (a+ 2+ 2a-t)(a? + 2-7 — 2a-1) fate 4 = (a? 2a 2)(a?- 2a + 2}. dl b) Para b= 5: Exemplo Resolvido 217: Efetue 16p* + 324q%. Resolucao: Podemos escrever: 16p' +324q4 = (2p) +4.(3q)* => ; 2 39) 16p4 +324q -[(207 +2(3q)" +2(2p)(3q) [(20)° +2(3q)? ~2(2e) +. 16p* + 324q4 = (4p? + 18q? +12pq)(4p? + 18q7 ~12pq). Exemplo Resolvido 218: Efetue 625m* +4. Resolugaéo: Podemos escrever: E=625m4+4 = E=(5m)* 44 > =| (6m)? +2-(5m)+-2]| (5m? -2.(6m)+2| + 625m* +4 = (25m? +40m +2)(25m? ~10m+2). Os Segredos da Algebra para IME/ITA/OLIMPIADAS 193 plo Resolvide 219: Prove que 1634 +7 (4a? 2-2} 4a2— 2044), 4 2 2 Exem| pesolusae: Fodemes escrever win 1. 1\ 16a #1 t=(2ai" a 16a4 “~; (2a)? +(2a)+5 | (ax)? ~(2a)+5 | seat «2-[ 40° +2a+3)( 4a? -21+3) 5.37) Uma Identidade Interessante_ Byiste uma identidade muito parecida com a identidade de Sophie-Germain, podemos dizer que € um caso particular, 0 seu raciocinio € 0 mesmo, mas ppenes um detalhe chama aten¢ao. E dada por: lat bt = (2° + abv2 + b?)(a?— abv2 + 62) Demonstracao: a +bt =a" +b4 420%? 2a?b? => at i b* = (a? +b?) - (abv)" ptcb* (a? aby -b?)( a? advo 0" Consequéncia: Para b = 1, temos: lat 1 = (2. a B + 1\(a?- av + 1) Demonstracao: +14 2a? - 2a? @ at +1= (a?+ iy - (a8) Bets (eave + at az +1) Ex Mplo Resolvido 220: Efetue Bix! +1= (9x2 + 3x2 +1)(9x? -S«02 +1). Reso) "uso: Podemos escrever: iene (3x) 41 = axd + 4=[ (90) + (9x) a8 +1] 87 9602 san 41229084). —(3x)- B+] 194 5 Produtos Notaveis 5.38) Identidade de Chrystal: - _ ‘Uma identidade diferente, que tem algumas aplicagdes em olimpiadas spa por: 2 —_—_ 1 ee +t (b-cy? (b? ~?) (d+o}? Demonstracao: (b of +brop+a(b? cf) (b-e)' +(bs of? + a(t? b4 — 2b2c? +c* (b?-c2 y (o-oF | (bso)? 20? -%) oe ep (0? --a)" (eayle pyre 7 1 *Ta-ay(d-by(d—«) b a : b , c >t ) bya-cand D=ayo- ob" (—ayle-oye a’ ¢—<— (d—a)(d—b)(d—c) 2 c2 ©) BBye—eyla= "aya "[e-ayle yong’ da *(eale-oy(e-e) a b? c {a-b)(a—c)(a-d) " (b-ay(b-ey(b-d) " (e-ay(c-byfe a)" a “(eaya-by(e=e) 3 dq) Questao 5.81 Sejam a, b ec numeros reais, tais que 1 + 3 + 1 = abc a+b+c ‘ 1.41.1 4 denominadores diferentes de zero. Prove que — += + -~- =_———z.. 55d od > ab? o (a+b+c) , com os Questdo 5.82 (Finlandia 2002) Mostre que se gibi canes entao. 41,1, 1__ tt deste abc a+bic’ a ph oh aha prec que n seja um inteiro positivo impar. Questao 5.83 2 4b Simplifique —\— ,— _ 2a-b b?-4a? arb Os Segredos da Algebra para IMEATAOLIMPIADAS 197 simplifique questao 5.95 (CN-1961) quest y x —_—-— e plifiqu ay yak Efetue e SIMI questo 5.86 (CN-1978) —ccar a*—p* para bz ta, simplificando —>——-5 5 _2ab_ mm (a? +b? + 2ab)(a +b? ~2ab) Fp obtemos: a+b 7 ab 5 : b att y “2 a Questao 5.87 )P _ (ay—bx? Mostre que / x+y xy(x+y) Questo 5.88 4 ant (nt) +4né (n+1) +4n' (n 1) +40 Mostre que o valor de <> z r+ (an+ty 2 +(2n-1) uy estéo 5.89 (Gra-Bretania-2014) - - 4 Selermine o valor de 20144 +4 20134 2012 24-2013 : 2013? + 40277 2013? + 4025 _ Ques ae a2 aa) € °5.90 (AIME-1987) ot 5 324)(224 +324)(34* +324 m Ms (qs *824)(194 +324)(264 +324)(40" + a4)(62 +324) Determine = Produtos Notaveis 198 5 (24s | } —— (44 ey Determine o valor de Tee ( a 4 Vfg4 ( 1 y— \ ny Os Segredos da Algebra para IME/TA/OLIMPIADAS 5 190 agora chegow © momento das identidades condicionais, \dentidades @ muito frequentes em ollmpiadas. Voramwon An ieniiindes 8 at Wdontidades import iantissimas 1as competigdes, elas aparece Como OxEICICION pare proves bastante atengo nas demonstragées! mais importantes tas, entao preste jidades Condicionais: 5,39) Ident 0, entdo temos: Demonstraco: (a +b +c) = at4b?4 67+ 2(ab 4 ac + be) => (0)? = a®+ b? + 67 + 2(ab 4 ac + be) a? + b2 4 c% = —2(ab 4 ac + be) b) Quadrado da Soma Dois a Dois: (ab)? + (be)* + (ac)? [ab + ac + be)? Demonstragao: (ab + be + ac)’ = ab)? + (be)? + (ac)? +2abe (a +b +6) ( (eb + be + ac)? = (ab)? + (be)? + (ae)? +2abe-(0) ab + ac + be)’ = (ab)? (bo)? + (ae)? Q ) Soma de Cubos Simples: fa? 1 b? 4c? = Sabe} Penonstracao: (asbyc3 cy =a8 +b340943(a+b+e (0° - ("= a4 by @%. a(a)(ab + ao + be )(ab +ac-+be) - 3abe )- 3abe lee. 53 b+ 03 = dabe].

Você também pode gostar